Physiology Pre-Test

Ace your homework & exams now with Quizwiz!

358. A 10-year-old boy presents with below-average body weight and height, signs of vitamin K deficiency, steatorrhea, and bloating. He is found to have the MHC class II antigen HLA-DQ2. Which of the following is the most appropriate dietary treatment for malabsorption in this condition? a. Fat-free diet b. Gluten-free diet c. High-fiber diet d. Lactose-free diet e. Low-salt diet

The answer is b

461. A 43-year-old man develops a brain tumor that impinges on the supraoptic nucleus in the hypothalamus. As a result, the secretion of which of the following hormones is affected? a. Adrenocorticotropic hormone (ACTH) b. Antidiuretic hormone (ADH) c. Follicle-stimulating hormone (FSH) d. Growth hormone e. Prolactin

The answer is b

463. A 39-year-old man with an enlarged head, hands, and feet; osteoarthritic vertebral changes; and hirsutism presents with a complaint of gynecomastia and lactation. The patient is most likely suffering from a tumor in which of the following locations? a. Adrenal cortex b. Anterior pituitary c. Breast d. Hypothalamus e. Posterior pituitary

The answer is b

108. A 42-year-old man sees his ophthalmologist because it is becoming increasingly difficult for him to read the newspaper. His vision problem most likely results from an inability to contract which of the following? a. Ciliary body b. Extraocular muscles c. Iris d. Pupils e. Suspensory ligaments

The answer is a

1. A 61-year-old man with erectile dysfunction asks his physician to prescribe Viagra® (sildenafil citrate; Pfizer Inc., Mission, KS, USA). Sildenafil produces its physiological effects by blocking the enzyme that hydrolyzes the second messenger by which nitric oxide produces its physiological effects. Which of the following is the second messenger? a. Cyclic guanosine monophosphate (cGMP) b. cGMP phosphodiesterase (PDE) c. Diacylglycerol d. G protein e. Guanylate cyclase

the answer is a

18. A 10-month-old, well-nourished, lethargic infant is brought to the emergency department with a history of vomiting and profuse watery diarrhea for 5 days. His mother reports that he has also had a marked decrease in urine output. Laboratory test results are as follows: The infant is treated for gastroenteritis and a saline drip is started. After 3 days, he appears well and alert and his diarrhea and vomiting have subsided. However, he still has hypernatremia, polyuria, and low urine sodium. These persistent clinical signs are most likely due to which of the following? a. Diabetes insipidus b. Diabetes mellitus c. Hypothyroidism d. Hyperaldosteronism e. Renal failure

the answer is a

10. A 43-year-old pregnant woman develops preeclampsia at 32 weeks' gestation. Intravenous labetalol is given to reduce blood pressure, and magnesium sulfate, which blocks N-methyl-D-aspartate (NMDA) receptors in the central nervous system, is ordered for the prevention of eclamptic seizures until the fetus can be delivered. Which of the following activates the NMDA receptor? a. Acetylcholine b. Gamma-aminobutyric acid (GABA) c. Glutamate d. Glycine e. Kainate

the answer is c

39. A 25-year-old man training for a 10-km race runs at a moderate level of approximately 25% of his maximal oxygen consumption. During the increase in aerobic metabolism in the exercising skeletal muscles, most of the volatile acid entering the blood is buffered by which of the following? a. Bicarbonate b. Hemoglobin c. Lactate d. Phosphates e. Plasma proteins

the answer is b

58. An 86-year-old man waiting to see his doctor is sitting in an examination room in which the air temperature is 21°C (70°F). He is wearing only a thin gown that is open in the back. The majority of his body heat will be lost by which of the following mechanisms? a. Breathing b. Cutaneous vasoconstriction c. Horripilation d. Radiation and conduction e. Vaporization of sweat

the answer is d

105. A 17-year-old boy is admitted to the hospital with a traumatic brain injury, sustained when he fell off his motorcycle. He develops a fever of 102.2°F (39°C), which is unrelated to an infection or inflammation. The fever is most likely due to a lesion of which of the following? a. Anterior hypothalamus b. Arcuate nucleus c. Lateral hypothalamus d. Paraventricular nucleus e. Posterior nucleus

The answer is a

109. At a first-grade parent-teacher conference, the teacher of a 6-year-old boy indicates that the boy seems to have difficulty hearing. His parents take him to the pediatrician, who refers the boy to an otolaryngologist. The boy is found to have a significant hearing deficit accompanying a middle ear infection that also involves the middle ear bones. Which of the following is the primary function of the middle ear bones? a. Amplifying sounds b. Enhancing frequency discrimination c. Filtering high-frequency sounds d. Localizing sound e. Protecting the ear from load sounds

The answer is a

114. A 20-year-old boxer presents at a neurologist's office complaining of dizziness and a problem with his balance. He indicates that in a recent match, he suffered several blows to the ears. Which of the following normally happens when a person slowly rotates toward the right? a. Both the left and right eyes deviate toward the left b. The endolymph in the left and right horizontal semicircular canals moves in opposite directions c. The hair cells in the left horizontal semicircular canal become depolarized d. The stereocilia on the hair cells in the right horizontal semicircular canal bend away from the kinocilium e. The visual image on the retina becomes unfocused

The answer is a

116. An 8-year-old boy is hit in the head by a baseline drive during a little league game. His father, a doctor, rushes out on the field to do a neurological assessment, including use of his penlight to check reactivity of the pupils. When light strikes the eye, which of the following normally increases? a. The activity of transducin b. The amount of transmitter released from the photoreceptors c. The concentration of all-trans retinal within the photoreceptors d. The concentration of calcium within the photoreceptors e. The activity of guanylyl cyclase

The answer is a

121. A 24-year-old male medical student develops apprehension, restlessness, tachycardia, and tachypnea as he enters the testing center for his initial licensure examination. Activation of which of the following receptors would be expected to decrease his anxiety? a. GABAA b. Glutamate c. Histamine d. Neurokinin 1 (NK-1, substance P) e. Neuropeptide Y

The answer is a

130. A 19-year-old woman with a history of diplopia and paresthesia is diagnosed with MS. Immersion of an affected limb in a cold bath restores nerve conduction in many MS patients. The explanation often cited for this effect is that cold increases the duration of the action potential. Which of the following best explains why increasing the duration of the action potential can restore nerve conduction in patients with MS? a. The amount of sodium entering the nerve with each action potential increases b. The capacitance of the nerve fiber membrane is increased c. The duration of the refractory period is increased d. The potassium conductance of the membrane is increased e. The membrane potential becomes more positive

The answer is a

133. A 52-year-old man presents at the oral surgeon's office with an abscessed tooth. Prior to surgery to extract the tooth, the patient is given a shot of procaine. Preventing the inactivation of sodium channels by local anesthetics will decrease which of the following? a. Downstroke velocity of nerve cell action potentials b. Duration of nerve cell action potentials c. Magnitude of the overshoot in nerve cell action potentials d. Relative refractory period of nerve cells e. Upstroke velocity of nerve cell action potentials

The answer is a

137. A 16-year-old, highly allergic girl who is stung by a bee gives herself a shot of epinephrine prescribed by her physician. Because epinephrine activates β-adrenergic receptors, it will relieve the effects of the bee sting by decreasing which of the following? a. Contraction of airway smooth muscle b. Rate of depolarization in the SA node c. Rate of glycogenolysis in the liver d. Strength of ventricular muscle contraction e. Transport of calcium into skeletal muscle fibers

The answer is a

144. A 47-year-old man presents with pain and swelling in the left big toe. He has been on a high-protein diet and training extensively for an iron man competition, which includes weight lifting, running, swimming, and biking. An aspirate of joint fluid demonstrates negative birefringent urate crystals and elevated 24-hour urinary uric acid levels. For maintenance therapy, which of the following agents would be best to treat his condition? a. Allopurinol b. Aspirin c. Colchicine d. Nonsteroidal anti-inflammatory drugs e. Uricosurics

The answer is a

149. A 12-year-old boy with muscular dystrophy is found to have a mutation of the gene that encodes the protein dystrophin. Genetic alterations in dystrophin lead to progressive muscular weakness because dystrophin provides structural support to the sarcolemma by binding which of the following? a. Actin to β-dystroglycan b. Actin to the Z lines c. β-Dystroglycan to laminin d. Z lines to M lines e. Z lines to the sarcolemma

The answer is a

163. A 68-year-old woman with pulmonary fibrosis presents with a complaint of increasing dyspnea while performing activities of daily living. She is referred for pulmonary function testing to assess the progression of her disease. Which of the following laboratory values is consistent with her diagnosis? a. Decreased diffusing capacity of the lung b. Increased residual volume c. Decreased forced expiratory volume exhaled in 1 second (FEV1)/forced vital capacity (FVC) d. Increased lung compliance e. Increased airway resistance corrected for lung volume

The answer is a

189. A person with CHF and progressive shortness of breath is admitted to the hospital for cardiac transplantation surgery. Hemodynamic recordings made with a Swan-Ganz catheter were as follows: Mean pulmonary artery pressure (PAP): 35 mm Hg Mean left atrial pressure (LAP): 20 mm Hg Pulmonary artery wedge pressure (PAWP): 25 mm Hg Cardiac Output: 3 L/min On a previous admission, the patient's LAP was 15 mm Hg and cardiac output was 4 L/min. What can be deduced from these data? a. Cardiac contractility is lower than on the previous admission. b. Left ventricular preload is lower than on his previous admission. c. Net fluid absorption into the pulmonary capillaries is increased. d. Pulmonary capillary hydrostatic pressure is lower than normal. e. Pulmonary vascular resistance is lower than normal at present.

The answer is a

190. A 68-year-old man with chronic obstructive pulmonary disease (COPD) entered the emergency department complaining of shortness of breath. His respirations were 35 per minute and labored. He had a productive cough and rales were heard over all lung fields. The patient had a rather ashen complexion and his nail beds gave clear evidence of cyanosis. An arterial blood sample was obtained and a chest x-ray was ordered. The patient was then placed on an O2 mask delivering 40% O2. One half hour later, the physician was called to the bedside by the nurse who found the patient unresponsive. The patient's complexion had changed to a flushed pink with no trace of cyanosis. His respirations were quiet at a rate of 6 per minute and a tidal volume of 300 mL. Repeat arterial blood gases showed that his arterial PCO2 had increased from 55 to 70 mm Hg, and his PaO2 increased from 55 to 70 mm Hg. Oxygen therapy most likely resulted in which of the following? a. Alveolar hypoventilation b. Elimination of the hypercapnic drive c. Hypoxic pulmonary vasoconstriction d. Increased firing of carotid body chemoreceptors e. Oxygen toxicity

The answer is a

198. A healthy, 24-year-old man is prescribed sustained-release bupropion (Zyban) for smoking cessation. Three weeks later, he presents to his family physician with intermittent fever and a generalized rash, at which time the bupropion is discontinued. A month later, he develops a dry, intermittent cough and dyspnea. Which of the following pulmonary function results is consistent with allergic bronchospasm? a. A decreased FEV1/FVC b. A decreased residual volume c. An increased diffusing capacity d. An increased FVC e. An increased lung compliance

The answer is a

202. A 65-year-old smoker develops a squamous cell bronchogenic carcinoma that metastasizes to the tracheobronchial and parasternal lymph nodes. The chest x-ray is consistent with accumulation of fluid in the pulmonary interstitial space. Flow of fluid through the lymphatic vessels will be decreased if there is an increase in which of the following? a. Capillary oncotic pressure b. Capillary permeability c. Capillary pressured. Central venous pressure e. Interstitial protein concentration

The answer is a

204. A 57-year-old woman presents with dyspnea on exertion. Pulmonary function studies with plethysmography demonstrate an increased resting oxygen consumption and work of breathing. Which of the following will decrease the oxygen consumption of the respiratory muscles? a. A decrease in airway resistance b. A decrease in diffusing capacity of the lung c. A decrease in lung compliance d. An increase in rate of respiration e. An increase in tidal volume

The answer is a

215. A healthy 32-year-old woman undergoes pulmonary exercise stress testing prior to starting a training regimen in preparation for her first marathon. Normally, during moderate aerobic exercise, which of the following occurs? a. Alveolar ventilation increases b. Arterial pH decreases c. Arterial lactate level increases d. PaCO2 decreases e. PaO2 increases

The answer is a

226. A 43-year-old woman with a history of asthma presents to the emergency department with an acute asthma attack after her bronchodilator inhaler ran out the day before. Airway resistance is greater at which of the following? a. At low lung volumes compared with high lung volumes b. At lower values for Reynolds number c. During inspiration compared with expiration d. In the total cross-section of the small airways compared with the total cross-section of the central airways e. With laminar flow than with turbulent flow

The answer is a

239. A 66-year-old man is referred to a cardiologist for evaluation. Physical examination reveals a diastolic murmur prominent over the left sternal border, a decrease in diastolic pressure, and an increase in pulse pressure. Of the following, which is the most likely diagnosis? a. Aortic regurgitation b. Aortic stenosis c. Mitral valve prolapse d. Pulmonary regurgitation e. Pulmonic stenosis

The answer is a

251. A 23-year-old woman presents with fatigue. She is found to have a mid-systolic murmur and higher-than-normal cardiac output. The differential diagnosis based on these findings includes which of the following? a. Anemia b. Cardiac tamponade c. Mitral regurgitation d. Mitral stenosis e. Third-degree heart block

The answer is a

252. A 22-year-old woman with lightheadedness and recurrent syncope is taken to the emergency department. Her symptoms are relieved by intravenous atropine administration. Antagonism of cholinergic muscarinic receptors causes an increase in which of the following physiologic variables? a. Heart rate b. PR interval c. Stroke volume d. TPR e. Ventricular contractility

The answer is a

256. A 64-year-old woman is postoperative day 1 after a cholecystectomy. She suddenly stands up after being supine since the operation. As a result, which of the following hemodynamic variables will increase? a. Heart rate b. Mean arterial pressure c. Stroke volume d. Venous return e. Ventricular end-diastolic volume

The answer is a

261. During auscultation of a patient with long-standing hypertension, the physician notes that the splitting of the second heart sound is reversed with P2 occurring before A2. Which of the following is a common ECG finding accompanying paradoxical splitting of the second heart sound? a. Left bundle branch block b. Right bundle branch block c. Sinus arrhythmia d. Sinus bradycardia e. Sinus tachycardia

The answer is a

266. A pacemaker is inserted in a patient in order to treat a prolonged PR interval detected on the ECG. Which of the following normally occurs during the PR interval? a. The cardiac action potential passes through the AV node b. The mitral and aortic valves are both closed c. There is no change in the voltage tracing on the ECG d. The second heart sound is heard e. The ventricles are contracting

The answer is a

278. A 37-year-old patient is brought to the emergency department in shock. A decision is made to treat anaphylactic shock rather than hypovolemic shock based on an increase in which of the following variables? a. Cardiac output b. Heart rate c. Serum creatinine d. Total peripheral resistance e. Ventricular contractility

The answer is a

283. A 47-year-old man with type II diabetes reports for his 6-month checkup. His doctor prescribes a daily 30-minute routine of walking at a brisk pace. During aerobic exercise, blood flow remains relatively constant to which of the following organs? a. Brain b. Heart c. Kidneys d. Skeletal muscle e. Skin

The answer is a

287. A 63-year-old woman presents to the emergency room with complaints of dyspnea, an elevated jugular venous pulse, and bilateral lower extremity edema. She is prescribed captopril. Which of the following best describes a beneficial effect of this drug? a. Afterload is decreased b. Arteriolar vasoconstriction is augmented c. Bradykinin is reduced d. A nonproductive cough can develop e. Survival is decreased

The answer is a

293. A 63-year-old woman presented with acute onset of right eye pain. Ophthalmic and neurologic examinations were normal except for a loud right carotid bruit. The eye pain ceased following carotid endarterectomy. The bruit was most likely caused by which of the following? a. High velocity of blood within the carotid artery b. Increase in blood viscosity c. Increase in hematocrit d. Lengthening of the carotid artery e. Widening of the carotid artery

The answer is a

301. During a clinical elective, a second-year medical student auscultates the heart of a patient, which reveals normal S1 and S2 heart sounds with no murmurs. In correlating the physical examination with the cardiac cycle, when does the highest coronary blood flow per gram of left ventricular myocardium occur? a. At the beginning of diastole b. At the beginning of isovolumic contraction c. When aortic blood flow is highest d. When aortic pressure is highest e. When left ventricular pressure is highest

The answer is a

307. A healthy 3-year-old girl presents for a health maintenance examination. Her developmental history is unremarkable and her immunizations are up to date. Upon cardiac auscultation, there are no murmurs, rubs, or gallops, and an S1 and an S2 are heard. S2 is split at a fixed interval and does not vary with respiration. Which of the following conditions is most likely present in this patient? a. Atrial septal defect b. Bicuspid aortic valve c. Coarctation of the aorta d. Dilated ventricles e. Normal cardiac variant

The answer is a

312. A 27-year-old female medical student with irritable bowel syndrome (IBS) has an alteration in intestinal motility resulting in fluctuating constipation and diarrhea. Her condition has worsened in the last month as the date she has scheduled for her licensure examination approaches. Which of the following best describes small intestinal motility? a. Contractile activity is initiated in response to bowel wall distention. b. Contractile frequency is constant from duodenum to terminal ileum. c. Migrating motor complexes (MMCs) occur during the digestive period. d. Peristalsis is the only contractile activity that occurs during feeding. e. Vagotomy abolishes contractile activity during the digestive period.

The answer is a

314. A 27-year-old woman comes to the emergency room because of a 2-day bout of profuse watery diarrhea. Physical examination reveals dry lips and oropharynx. The patient is diagnosed with acute secretory diarrhea and dehydration, likely due to Escherichia coli. Which of the following sodium reabsorptive pathways is inhibited by the enterotoxin? a. Electroneutral NaCl transport b. Sodium-bile salt cotransport c. Sodium-glucose coupled cotransport d. Sodium-hydrogen countertransport e. Sodium-phosphorous countertransport

The answer is a

315. A 37-year-old man presents with dehydration and hypokalemic metabolic acidosis. This acid-base and electrolyte disorder can occur with excess fluid loss from which of the following organs? a. Colon b. Duodenum c. Liver d. Pancreas e. Stomach

The answer is a

317. A 42-year-old man develops a gastric carcinoma affecting the proximal third of his stomach. He is scheduled for a partial gastrectomy of the affected region. Which of the following processes will be most affected by the surgery? a. Receptive relaxation b. Peristalsis c. Retropulsion d. Segmentation e. Trituration

The answer is a

326. A 43-year-old woman presents with dysphagia to solids and liquids, bland regurgitation, and diffuse chest pain of 2 months duration. During this time, she has lost about 20 pounds. The patient is scheduled for esophageal imaging and motility testing. The esophagogram during a barium swallow shows a dilated esophagus with an area of distal stenosis and esophageal manometry tracings during a wet swallow shows a high lower esophageal sphincter (LES) opening pressure and uncoordinated peristalsis. These findings are consistent with which of the following diagnoses? a. Achalasia b. Diffuse esophageal spasm c. GERD d. Schatzki ring e. Zenker diverticulum

The answer is a

332. A 68-year-old woman with rheumatoid arthritis, who has been taking nonsteroidal anti-inflammatory drugs (NSAIDs) for the past 10 years, complains of burning epigastric pain that is relieved by antacids, but worsened with food. Her doctor discontinues the NSAIDs and recommends cimetidine (Tagamet) because it is inexpensive and over the counter. Which of the following best describes the pharmacological blockade of histamine H2 receptors in the gastric mucosa? a. It inhibits both gastrin-and acetylcholine-mediated secretion of acid.b. It inhibits gastrin-induced but not meal-stimulated secretion of acid. c. It has no effect on either gastrin-induced or meal-stimulated secretion of acid. d. It prevents activation of adenyl cyclase by gastrin. e. It causes an increase in potassium transport by gastric parietal cells.

The answer is a

333. A 37-year-old man is admitted to the hospital due to an exacerbation of his Crohn's disease with severe inflammation of the ileum. Which of the following would be seen? a. Decreased bile acid pool size b. Decreased release of secretin c. Increased absorption of dietary fats d. Increased colon absorption of water e. Increased vitamin B12 absorption

The answer is a

335. A 57-year-old man undergoes resection of the distal 100 cm of the terminal ileum as part of treatment for Crohn's disease. The patient likely will develop malabsorption of which of the following? a. Bile salts b. Folate c. Iron d. Lactose e. Protein

The answer is a

343. A 26-year-old man presents to the emergency room with a 48-hour bout of diarrhea with steatorrhea. Which of the following best accounts for the appearance of excess fat in the stool? a. Decreased bile acid pool size b. Increased bile salt reabsorption c. Increased colonic microbiota d. Increased chylomicron formation e. Increased lipase secretion

The answer is a

345. An 83-year-old woman with constipation is prescribed a high-fiber diet, which leads to an increased production of short-chain fatty acids (SCFAs). SCFA absorption occurs almost exclusively from which of the following segments of the GI tract? a. Colon b. Duodenum c. Ileum d. Jejunum e. Stomach

The answer is a

360. A morbidly obese man presents with hypertension, hyperlipidemia, and type 2 diabetes mellitus. Dietary fat, after being processed, is extruded from the mucosal cells of the GI tract into the lymphatic ducts in the form of which of the following? a. Chylomicrons b. Free fatty acids c. Diglycerides d. Monoglycerides e. Triglycerides

The answer is a

366. A 33-year-old woman who has been taking large doses of NSAIDs for her menstrual cramps presents with burning epigastric pain that is worse after eating. Which of the following is the major factor that protects the duodenal mucosa from damage by gastric acid? a. Bicarbonate contained in bile b. Duodenal bicarbonate secretion c. Endogenous mucosal barrier of the duodenum d. Hepatic bicarbonate secretion e. Pancreatic bicarbonate secretion

The answer is a

381. A 54-year-old man with small cell lung cancer presents with lethargy, confusion, and muscle cramps. Blood work shows an increase in plasma levels of antidiuretic hormone (ADH). In patients with the syndrome of inappropriate antidiuretic hormone (SIADH), which of the following will increase? a. Intracellular volume b. Plasma oncotic pressure c. Plasma osmolarity d. Plasma sodium concentration e. Urine flow

The answer is a

384. A 63-year-old woman is brought to the emergency department complaining of fatigue and headaches. She appears confused and apathetic. She has been taking diuretics to treat her hypertension and paroxetine for her depression. Laboratory results are as follows: Urine flow = 1 L/day Plasma sodium = 125 mmol/L Plasma potassium = 4 mmol/L Urine osmolality = 385 mOsm/L Urine sodium = 125 mmol/L Urine potassium = 25 mmol/L Which of the following is the patient's approximate free water clearance? a. −0.20 L/day b. −0.50 L/day c. −0.75 L/day d. +0.2 L/day e. +0.50 L/day

The answer is a

385. A 28-year-old woman with systemic lupus erythematosus is brought to the emergency department after developing hypokalemic paralysis. Arterial blood gas analysis shows a Pa O2 of 102 mm Hg and a pH of 7.1. She is diagnosed with type I renal tubular acidosis (RTA) caused by an autoimmune response that damages the H+-ATPase on the distal nephron. Which of the following laboratory measurements will most likely be normal in this patient? a. Anion gap b. Aldosterone secretion c. Net acid excretion d. Serum bicarbonate e. Urine ammonium

The answer is a

386. A 24-year-old man with a history of renal insufficiency is admitted to the hospital after taking a large amount of ibuprofen. His BUN is 150 mg/dL. This patient's high serum urea nitrogen was most likely caused by which of the following? a. Decreased GFR b. Decreased secretion of urea by the distal tubules c. Increased reabsorption of urea by the proximal tubules d. Increased renal blood flow (RBF) e. Increased synthesis of urea by the liver

The answer is a

393. A 63-year-old hospitalized woman becomes oliguric and confused. A blood sample is drawn to measure her glucose concentration, which is found to be 35 mg/dL. An IV access is obtained and an ampule of 50% dextrose is given followed by a continuous infusion of 10% dextrose. Most of the glucose that is filtered through the glomerulus undergoes reabsorption in which of the following areas of the nephron? a. Proximal tubule b. Descending limb of the loop of Henle c. Ascending limb of the loop of Henle d. Distal tubule e. Collecting duct

The answer is a

395. A 64-year-old elementary school teacher complains of a strong sense of urinary urgency followed by incontinence of large amounts of urine as she tries to rush to the bathroom. She also states that she has had urinary frequency as well as nocturia over the last week. Her past medical history is insignificant, but she was recently diagnosed with a urinary tract infection last week. The patient is diagnosed with an overactive bladder with urinary incontinence. She is treated with behavioral training and oxybutynin. What neurotransmitter is responsible for initiating bladder (detrusor) contraction? a. Acetylcholine b. Epinephrine c. GABA d. Norepinephrine e. Serotonin

The answer is a

402. A 44-year-old woman presents with abdominal pain, fever, and chills. Physical examination reveals costovertebral angle tenderness, previously undiagnosed hypertension, and a mid-systolic click. Urine culture shows bacteriuria and free water clearance is positive, indicating excretion of dilute urine. The ability of the kidney to excrete concentrated urine will increase if which of the following occurs? a. The activity of the Na+-K+ pump in the loop of Henle increases. b. The flow of filtrate through the loop of Henle increases. c. The glomerular capillary pressure increases. d. The permeability of the collecting duct to water decreases. e. The reabsorption of Na+ by the proximal tubule decreases.

The answer is a

409. A patient with chronic obstructive lung disease and cor pulmonale is given acetazolamide to inhibit carbonic anhydrase, along with a thiazide diuretic. How does the distal nephron differ functionally from the proximal tubule? a. The distal nephron has a more negative intraluminal potential than the proximal tubule. b. The distal nephron is less responsive to aldosterone than the proximal tubule. c. The distal nephron is more permeable to hydrogen ion than the proximal tubule. d. The distal nephron secretes more hydrogen ion than the proximal tubule does. e. The distal nephron secretes less potassium than the proximal tubule does.

The answer is a

410. An 82-year-old man who recently suffered head trauma from a car accident presents with polyuria and polydipsia. Blood analysis reveals hypernatremia and urinalysis shows hypotonicity and an increased free water clearance. In which of the following conditions is an increased free water clearance a hallmark of the disease? a. Diabetes insipidus b. Diabetes mellitus c. Diuretic therapy d. Heart failure e. Renal failure

The answer is a

416. A 29-year-old man expresses concern about his upcoming skiing trip to Breckenridge, Colorado (elevation = 10,000 ft). He states that every time he goes there, he gets high altitude sickness that is relieved when he is given oxygen. The family physician gives the patient a prescription for oxygen to use when he arrives in Colorado, as well as a prescription for acetazolamide to take for 2 days prior and throughout his 4-day trip. Carbonic anhydrase inhibitors exert their diuretic effect by inhibiting the reabsorption of Na+ in which of the following parts of the nephron? a. The proximal tubule b. The thick ascending limb of loop of Henle c. The distal convoluted tubule d. The cortical collecting duct e. The outer medullary collecting duct

The answer is a

424. An elderly woman presents with spiking fever, shaking chills, nausea, and costovertebral angle tenderness. Urine cultures are positive and she is hospitalized for pyelonephritis. Her glomerular filtration rate (GFR) decreases with a resultant increase in the concentration of NaCl delivered in the intraluminal fluid to the thick ascending limb of the loop of Henle. Under these conditions, the macula densa will increase the formation and release of which of the following substances? a. Adenosine b. Aldosterone c. Angiotensinogen d. Antidiuretic hormone (ADH) e. Renin

The answer is a

432. A hypertensive patient develops chronic renal failure from progressive nephrosclerosis. Which of the following is associated with chronic renal failure? a. A decrease in the excretion of creatinine b. A decrease in the fractional excretion of sodium c. A decrease in net acid excretion d. An increase in free water clearance e. A normal anion gap

The answer is a

443. An 18-year-old college woman is brought to the emergency department by her roommate after she was raped walking back to the dorm from the library at night. She requests the "morning after pill" she has heard about to prevent pregnancy from the violation. She is given a postcoital contraceptive to prevent implantation and induce regression of the corpus luteum. Which of the following best describes implantation of the zygote in the uterine wall? a. Involves infiltration of the endometrium by the syncytiotrophoblast b. Is inhibited by secretion of progesterone from the corpus luteum c. Occurs 3 to 5 days after fertilization d. Occurs when the embryo consists of approximately 128 cells e. Precedes formation of the zona pellucida

The answer is a

453. A 29-year-old woman delivers a 7-lb 6-oz baby girl without complication. She begins to produce and eject breast milk a few days later. Prolactin secretion is tonically suppressed in nonpregnant women by which of the following hormones? a. Dopamine b. Estrogen c. FSH d. LH e. Progesterone

The answer is a

454. A young couple has been trying to conceive a baby. The medical director of the fertility center has advised the woman to take her basal temperature readings on a daily basis and for them to have intercourse at the time the woman appears to be ovulating. Once conception takes place, which of the following must occur in order for the pregnancy to proceed uneventfully? a. The corpus luteum must secrete progesterone to sustain the endometrium. b. The pituitary must secrete hCG to maintain the corpus luteum. c. The pituitary must secrete prolactin to sustain the placenta. d. The placenta must secrete FSH to maintain ovarian function. e. The placenta must secrete LH to maintain ovarian function.

The answer is a

458. A 35-year-old woman presents to her obstetrician/gynecologist's office for her annual well-woman examination. She reports that she may have "a touch of the flu" because she has been tired and nauseated the past week and also has had fleeting episodes of lower abdominal pain. She wasn't sure when her last menstrual period started but, after looking at a calendar, realized that it had been 38 days. A right adnexal mass was palpated on routine pelvic examination and subsequently observed on ultrasound. Urinalysis confirmed that the woman was pregnant and serum levels of the tumor marker CA-125 were elevated. The gynecologist informed the woman that she may have an ovarian cancer and may need her ovary removed, but that they need to wait another week or two to do the laparotomy in order to protect her fetus. Ovariectomy before the 6th week of pregnancy leads to abortion, but thereafter has no effect on pregnancy because the placenta secretes adequate amounts of which of the following hormones? a. Estrogens and progesterone b. Estrogen and relaxin c. Growth hormone-releasing hormone and corticotropin-releasing hormone d. Human chorionic somatomammotropin and hCG e. Progesterone and hCG

The answer is a

459. A 54-year-old man is prescribed finasteride, a 5-alpha reductase inhibitor, for his benign prostatic hypertrophy. Why is his pregnant wife instructed to not even handle the medication? a. Blocking the production of dihydrotestosterone will interfere with normal sexual differentiation of the penis, scrotum, and prostate in male fetuses. b. Inhibiting 5-alpha reductase in utero will impair SRY gene expression and consequently development of the testes. c. Inhibiting 5-alpha reductase in utero will lead to precocious puberty in both sexes. d. Inhibiting 5-alpha reductase in utero impairs secretion of MIS. e. Inhibiting 5-alpha reductase in utero will cause regression of the Wolffian ducts such that a male fetus develops female genitalia.

The answer is a

462. Following neck surgery, a patient develops circumoral paresthesia and a long QT interval on the electrocardiogram consistent with hypocalcemia resulting from injury to the parathyroid glands. Which of the following best describes parathyroid hormone (PTH)? a. It acts directly on bone cells to increase Ca2+ resorption and mobilize Ca2+. b. It acts directly on intestinal cells to increase Ca2+ absorption. c. It is synthesized and secreted from the oxyphil cells in the parathyroid glands. d. It increases phosphate reabsorption in the renal proximal tubular cells. e. Its secretion is increased in response to an increase in plasma-free Ca2+ concentration

The answer is a

471. A 35-year-old woman presents to her primary care physician with weakness and fatigue for the past 6 months. Before this current episode, she used to be a very active runner, but has not had the strength or energy to work out in the last few weeks. Despite this decrease in exercise, she has lost 10 lb over this time, and also reports a decreased appetite. She has also had two presyncopal episodes in the last 2 weeks and has noticed that her skin appears darker than usual. Laboratory tests show: Sodium: 125 Potassium: 5.5 Renin: Elevated Aldosterone: Suppressed Serum ACTH: >100 pg/mL ACTH stimulation test: Cortisol 10 µg/dL What is the best description of the patient's fluid and osmolarity status? a. Hyposmotic volume contraction b. Hyposmotic volume expansion c. Hyperosmotic volume contraction d. Hyperosmotic volume expansion e. Isosmotic volume contraction

The answer is a

472. A 3-year-old patient with DiGeorge congenital thymic aplasia presents with a seizure. An elevated serum phosphorus and low serum calcium confirm a hypoparathyroid state. Plasma levels of calcium can be increased most rapidly by the direct action of PTH on which of the following? a. Bones b. Intestine c. Kidney d. Skeletal musculature e. Thyroid gland

The answer is a

490. A 13-year-old girl presents for her annual sports physical exam. Her height is measured at 50 in (>3 SD below the mean for her age), and the history suggests that the girl may be suffering from anorexia nervosa. Which of the following about growth and development is most likely? a. Growth hormone activates the JAK2-STAT pathway. b. Linear growth ceases earlier in boys than in girls. c. Serum IGF-I levels decrease throughout childhood. d. Growth hormone is essential for prenatal linear growth. e. Normal growth during puberty is independent of thyroid function

The answer is a

65. A 24-year-old woman presents to her family physician with intractable hiccups. The patient is instructed to breathe into and out of a bag in order to rebreathe exhaled CO2. In the blood, the majority of CO2 is transported as which of the following forms? a. Bicarbonate b. Carbaminohemoglobin c. Carbonic acid d. Carboxyhemoglobin e. Dissolved CO2

The answer is a

67. A 37-year-old man presents with low exercise tolerance. Blood work shows a normal hematocrit and Hb concentration but a decreased P50. Which would be true of his oxyhemoglobin transport and dissociation? a. Hb's affinity for oxygen is increased b. O2 loading at the alveolar-capillary level is less than normal c. O2 saturation is lower than normal at any PaO2 d. O2 unloading is increased at the tissue level e. The differential diagnosis includes a point mutation resulting in increased binding of H+ to his Hb chains.

The answer is a

68. A 35-year-old woman presents to her family physician's office with fatigue of at least 3 months' duration. Her only explanation is that keeping up with her twin 4-year-olds really tires her out, and she does not even have enough energy to make nutritious meals for her and her husband. Vital signs and ECG are normal, but a third heart sound is heard with auscultation and she is pale. Blood results are as follows: Hb, 8 g/dL; hematocrit, 30%; MCV, 115 fL; WBC, 8000/μL; platelets, 200,000/μL. A deficiency of which of the following substances can most likely account for these findings? a. Folate b. Glucose-6-phosphatase c. Iron d. Niacin e. Zinc

The answer is a

76. A 32-year-old woman presents to the emergency department with a chief complaint of acute shortness of breath and right-sided chest pain, which increases during inspiration. She does not have a cough or fever, and does not have a history of asthma or other respiratory disease. She has not been ill or immobile, but reports having taken oral contraceptives for 8 years until shortly before conceiving her first child about 2 years ago. The family history is notable for her mother who died of a pulmonary embolism. Her respiratory rate is 25 breaths/min and her heart rate is 110 beats/min. Chest x-ray is normal, but a ventilation/perfusion scan reveals a possible pulmonary embolism. Which of the following blood disorders is associated with a hypercoagulable state? a. Activated protein C resistance b. Antithrombin III (AT-III) excess c. Disseminated intravascular coagulation (DIC) d. Hypoprothrombinemia e. Idiopathic thrombocytopenic purpura

The answer is a

82. A 16-year-old adolescent boy is brought to the emergency room by ambulance after suffering a concussion during a football game. When he awoke, he was able to understand and follow commands, including repeating language spoken to him, but he had difficulty with spelling, mild word-finding difficulty, and difficulty understanding written language and pictures. His condition is most likely caused by damage to which of the following? a. Angular gyrus in the categorical hemisphere b. Arcuate fasciculus connecting Broca and Wernicke areas c. Broca area in the frontal lobe d. Hippocampus e. Wernicke area at the posterior end of the superior temporal gyrus

The answer is a

93. A 16-year-old adolescent girl with epilepsy has an EEG recording done during a routine visit to her neurologist. The α-rhythm appearing on an EEG has which of the following characteristics? a. It disappears when a patient's eyes open b. It is associated with deep sleep c. It is replaced by slower, larger waves during REM sleep d. It produces 20 to 30 waves per second e. It represents activity that is most pronounced in the frontal region of the brain

The answer is a

98. A 22-year-old woman presents at the student medical center with tachycardia and palpitations. She reports that she has been taking a diet supplement containing ephedrine that she purchased from an Internet site. Activation of the sympathetic nervous system by ephedrine causes smooth muscle contraction in which of the following? a. Arterioles b. Bronchioles c. Ciliary bodies d. Intestines e. Pupils

The answer is a

104. A 24-year-old man complains of fatigue, increased daytime somnolence, and periodic sudden loss of muscle tone. Polysomnography confirms the diagnosis of narcolepsy. Narcolepsy is associated with which of the following? a. Decreased adenosine levels in the reticular formation b. Hypothalamic dysfunction with decreased CSF levels of orexins c. Increased discharge of noradrenergic neurons in the locus ceruleus d. Increased discharge of serotonergic neurons in the midbrain Raphé e. The presence of prions

The answer is b

111. A 62-year-old man with a history of hypertension and hyperlipidemia is admitted to the hospital for evaluation after demonstrating signs and symptoms of a stroke. Subsequent CT scans, perceptual tests, and a neurological examination provide evidence for impairment of the otolith pathways. The otolith organs (utricle and saccule) are responsible for which of the following? a. Detecting angular acceleration b. Detecting the position of the head in space c. Producing rotary nystagmus d. Producing the stretch reflex e. Producing the vestibulo-ocular reflex

The answer is b

123. After sitting with one leg crossed under the other for several hours while working on a document at her computer terminal, a 52-year-old woman tries to stand up, but is unable to walk on the crossed leg, and feels tingling and pain. Which of the following explains the loss of motor function without the loss of pain sensation in the peripheral nerves? a. A fibers are more susceptible to local anesthetics than C fibers b. Aβ fibers are more sensitive to pressure than C fibers c. C fibers are more sensitive to pressure than Aβ fibers d. C fibers are more susceptible to hypoxia than B fibers e. C fibers have higher conduction velocities than A fibers

The answer is b

127. An 80-year-old farmer presented with complaints of weakness and fatigue, aching, orthostatic hypotension, constipation, and sleep disturbances. His family physician told him that he was just getting old, and would have to get used to it. His bradykinesia worsened and he couldn't pick up his feet when he walked. When he was no longer able to plow his own fields, he got depressed, and his wife said he would just sit at the table and rub his thumb along his fingers. She called her son-in- law, a neurologist, and asked him if he'd come out to the country to evaluate "Pops." The pathophysiology of Parkinson disease can be attributed to a paucity of which of the following neurotransmitters? a. Acetylcholine b. Dopamine c. Glutamate d. Neuropeptide Y e. Serotonin

The answer is b

128. A 62-year-old man with COPD presents to the emergency room in respiratory distress. The attending physician uses succinylcholine to produce skeletal muscle relaxation prior to tracheal intubation. Soon after infusion of the succinylcholine, the patient develops a severe bradycardia. Which of the following drugs would counteract the bradycardia without affecting muscle relaxation? a. Acetylcholine b. Atropine c. Curare d. Dopamine e. Epinephrine

The answer is b

132. A 65-year-old postgastrectomy patient presents to his gastroenterologist's office with fatigue, weakness in his legs, and frequent falls over the past several months. His physical examination demonstrates increased deep tendon reflexes and decreased vibratory sense in his toes. Laboratory analysis reveals megaloblastic anemia and vitamin B12 deficiency. Which of the following mechanisms cause the neurological deficits characteristic of vitamin B12 deficiency? a. Decreased folate concentration b. Decreased myelin synthesis c. Decreased Na+-K+pump activity d. Increased hyperphosphorylated microtubule protein tau e. Production of antinerve antibodies (ANA)

The answer is b

140. A 56-year-old woman presents with a flat red rash on the face and upper trunk and an erythematous rash on the knuckles and in the shape of a V on the neck and anterior chest. She also complains of muscle weakness with difficulty getting out of a chair and combing her hair. Laboratory findings include increased creatine phosphokinase, positive antinuclear antibody, and anti-Mi2 dermatomyositis-specific antibodies. In addition to oral prednisone, physical therapy is ordered with the goal of improving muscle strength, and thus ability to perform activities of daily living. How does repetitive stimulation of a skeletal muscle fiber cause an increase in contractile strength? a. By increasing the concentration of calcium in the myoplasm b. By increasing the duration of cross-bridge cycling c. By increasing the magnitude of the end-plate potential d. By increasing the number of muscle myofibrils generating tension e. By increasing the velocity of muscle contraction

The answer is b

145. A 35-year-old woman presents at her ophthalmologist's office with a chief complaint of bilateral drooping eyelids. A complete history reveals that the woman has also been experiencing generalized fatigue and weakness that only improves if she takes frequent naps. The ophthalmologist suspects a synaptogenic basis for the ptosis and refers her to a neurologist. The patient is found to have circulating antibodies to nicotinic acetylcho-line receptors on the motor endplate. A drug is administered that increases the force of muscle contraction but causes bradycardia in the patient. What is the most likely mechanism of action of the drug? a. Decreases the concentration of calcium in the extracellular fluid b. Decreases the metabolic breakdown of acetylcholine c. Increases α-motoneuron discharge rate d. Increases the affinity of the skeletal muscle acetylcholine receptors to acetylcholine e. Increases the amount of acetylcholine released by α-motoneurons

The answer is b

153. A 5-year-old boy presents with abnormal running, jumping, and hopping. His parents have observed that he uses his arms to climb up his legs when rising from the floor. The pediatrician suspects Duchenne muscular dystrophy, and electromyography confirms a myopathy. The amount of force produced by a skeletal muscle can be increased by which of the following? a. Decreasing extracellular Ca2+ b. Decreasing the interval between contractions c. Increasing the activity of acetylcholine esterase d. Increasing extracellular Mg 2+ e. Increasing the preload beyond 2.2 mm

The answer is b

173. A 58-year-old woman experiences an acute exacerbation of asthma, which causes her breathing to become labored and faster. As a result, which of the following changes in airflow is expected? a. Flow in the trachea and upper airways will become more laminar. b. The pressure gradient required for airflow will increase. c. The resistance to airflow will decrease. d. The resistance to airflow will increase linearly with the decrease in airway radius. e. Reynolds number will decrease.

The answer is b

177. A 14-year-old adolescent girl presents with a lump in the neck. Fine needle aspiration biopsy reveals acinic cell carcinoma of the parotid gland. During the pariodectomy, there is compression injury of the glossopharyngeal nerve. As a result, which of the following respiratory reflexes will be impaired? a. Aortic baroreceptor reflex b. Carotid body chemoreceptor reflex c. Hering-Breuer inflation reflex d. Irritant airway reflex e. Juxta pulmonary capillary (J) receptor reflex

The answer is b

192. A 42-week gestation infant is delivered by cesarean section. Which of the following occurs with the baby's first diaphragmatic respiration? a. All of the fetal vascular channels functionally close. b. PaO2 increases. c. Pulmonary capillary hydrostatic pressure increases. d. Pulmonary vascular resistance increases. e. Systemic vascular resistance decreases

The answer is b

193. A 29-year-old woman is admitted to the hospital because of increasing dyspnea and swelling of both feet. An examination of her chest shows a severe pectus excavatum with only 2 cm of space between the vertebral bodies and the sternum. Pulmonary function tests show FVC and FEV1/FVC values that were 15% and 100%, respectively, of predicted. Which of the following laboratory measurements will most likely be below normal in this patient? a. Arterial PCO2 b. Arterial pH c. Elastic recoil of the chest wall d. Hemoglobin concentration e. Plasma bicarbonate concentration

The answer is b

232. A patient presents to the emergency department with intermittent chest pain. The ECG and blood tests are negative for myocardial infarction, but the echocardiogram shows thickening of the left ventricular muscle and narrowing of the aortic valve. An afterload-reducing medication is prescribed. Which of the following values would provide the best measure of the effectiveness of the medication in reducing left ventricular afterload in aortic stenosis? a. Left ventricular end-diastolic pressure b. Left ventricular mean systolic pressure c. Mean arterial blood pressure d. Pulmonary capillary wedge pressure e. Total peripheral resistance

The answer is b

247. A 38-year-old man has a murmur that ceases with the onset of the second heart sound. The second heart sound occurs at the onset of which phase of the cardiac cycle? a. Isovolumetric contraction b. Isovolumetric relaxation c. Rapid ejection d. Rapid ventricular filling e. Systole

The answer is b

255. A 50-year-old woman complains of intermittent chest discomfort. She is given an exercise stress test to determine if the angina is a result of myocardial ischemia. The test will be considered positive if which of the following occurs? a. An increase in mean arterial pressure b. Depression of the ST-segment c. Tachycardia d. A diastolic murmur e. Widening of the QRS complex

The answer is b

264. A 75-year-old woman makes an appointment to see her physician because of exertional dyspnea and an episode of syncope while dancing with her husband at their granddaughter's wedding. A systolic ejection murmur is auscultated that radiates to the carotid arteries. Her signs and symptoms are most likely due to which of the following? a. Aortic regurgitation b. Aortic stenosis c. Mitral stenosis d. Pulmonic regurgitation e. Tricuspid stenosis

The answer is b

274. During a routine physical examination, a 35-year-old man is found to have a blood pressure of 170/105 mm Hg. The history reveals episodes of headache accompanied by palpitations, diaphoresis, and anxiety. Which of the following is the best initial pharmacotherapy for this patient's most likely diagnosis? a. α-Adrenergic agonist b. α-Adrenergic antagonist c. β-Adrenergic agonist d. β-Adrenergic antagonist e. Glycoprotein IIb/IIIa antagonist

The answer is b

285. A 56-year-old man presents with complaints of fatigue and headaches. During physical examination, he is found to have a wide pulse pressure. Which of the following conditions causes pulse pressure to increase? a. Aortic stenosis b. Arteriosclerosis c. Heart failure d. Hemorrhage e. Tachycardia

The answer is b

289. An EMT arrives at the scene of an automobile accident, and finds a hemorrhaging, unconscious young woman. Which of the following is a sign of hemorrhagic shock? a. Bradycardia b. Low hematocrit c. Metabolic alkalosis d. Polyuriae. Warm skin

The answer is b

295. A 57-year-old woman is undergoing a femoral popliteal bypass for her peripheral vascular disease. The vascular surgeon wishes to induce a localized arteriolar constriction to help control hemostasis. An increase in the local concentration of which of the following agents will cause systemic vasoconstriction? a. Adenosine b. Antidiuretic hormone c. Atrial natriuretic peptide d. β2-Adrenergic agonist e. Nitric oxide

The answer is b

297. A 32-year-old man is diagnosed with primary hypertension. His physician recommends a drug for hypertension that acts by decreasing vascular smooth muscle contractile activity without affecting ventricular contractility. Which of the following is the most likely site of action for the drug? a. β-Adrenergic receptors b. Calmodulin c. Troponin d. Tropomyosin e. Protein kinase A

The answer is b

305. A 58-year-old man with a history of several months of exertional chest pain presents for evaluation. He also has severe arthritis in his knees bilaterally and is unable to undergo exercise stress testing. Thus, you elect to conduct a chemical stress test with dipyridamole to investigate the nature of his chest pain. Momentarily after administering the drug he begins to experience severe retrosternal chest pain and ST-segment depression in the anterior leads of the ECG. What is the most likely mechanism of this patient's chest pain? a. Bradycardia b. Coronary blood redistribution c. Coronary vasospasm d. Occlusive coronary embolus e. Pulmonary embolism

The answer is b

311. A 52-year-old man with diabetes mellitus type 1 has persistent nausea and vomiting due to gastroparesis with gastroesophageal reflux disease (GERD). Which of the following best describes the function of gastric emptying? a. Acidification of the antrum increases gastric emptying. b. Hyperosmolality of duodenal contents initiates a decrease in gastric emptying. c. Meals containing fat empty faster than carbohydrate-rich food. d. Solids empty more rapidly than liquids. e. Vagal stimulation decreases receptive relaxation in the upper portion of the stomach.

The answer is b

319. A 63-year-old woman has an intractable duodenal ulcer failing all previous treatments. After consultation with a surgeon, a laparoscopic vagotomy is performed. Subsequently, the patient experiences nausea and vomiting after ingestion of a mixed meal. Which of the following best explains her symptoms? a. Decreased gastric emptying of liquids b. Decreased gastric emptying of solids c. Increased gastric emptying of liquids d. Increased gastric emptying of solids e. Increased gastric emptying of solids and liquids

The answer is b

321. A 23-year-old woman complains of abdominal cramps and bloating that are relieved by defecation. Subsequent clinical evaluation reveals an increased maximal acid output, decreased serum calcium and iron concentrations, and microcytic anemia. Inflammation in which area of the GI tract best explains these findings? a. Stomach b. Duodenum c. Jejunum d. Ileum e. Colon

The answer is b

324. After a recent viral illness, a 20-year-old woman develops bilateral facial swelling consistent with parotitis. Which of the following best describes the salivary glands? a. Approximately 4 L of saliva is secreted per day. b. Salivary α-amylase preferentially hydrolyzes 1:6α over 1:4α linkages. c. Cranial nerve VIII passes through the parotid gland. d. Starch digestion begins in the mouth via salivary α-amylase. e. The parotid gland is the most mucinous of the salivary glands.

The answer is b

327. A 42-year-old salesman presents with the chief complaint of intermittent mid-epigastric pain that is relieved by antacids or eating. Gastric analysis reveals that basal and maximal acid outputs exceed normal values. The gastric acid hypersecretion can be explained by an increase in the plasma concentration of which of the following? a. Cholecystokinin b. Gastrin c. Secretin d. Somatostatin e. Vasoactive intestinal peptide

The answer is b

337. An 18-year-old college student reports that she experiences severe abdominal bloating and diarrhea within 1 hour of consuming dairy products. A subsequent hydrogen breath test is abnormal. The diarrhea and bloating can best be explained by which of the following? a. A decrease in exocrine pancreatic secretion b. A deficiency in the brush border enzyme lactase c. Carbohydrate-induced secretory diarrhea d. Decreased carbohydrate absorption e. Decreased intestinal surface area

The answer is b

340. A 20-year-old woman with type 1 diabetes mellitus presents with symptomatic hyperglycemia. The patient is started on insulin. The metabolic effects of insulin include which of the following? a. Decreased glucose utilization b. Decreased lipolysis c. Increased proteolysis d. Increased gluconeogenesis e. Increased ketogenesis

The answer is b

344. A 14-year-old ballerina reports that she has chronic diarrhea. A detailed history reveals that she frequently drinks skim milk, she does not use laxatives, and she has noticed that her condition improves when she fasts for religious observances. In contrast to secretory diarrhea, which of the following is most likely seen with osmotic diarrhea? a. It is caused by bacterial toxins. b. It is characterized by an increase in the stool osmotic gap. c. It is the result of decreased electroneutral sodium absorption. d. It is the result of increased crypt cell secretion. e. It occurs only in the colon

The answer is b

346. A 42-year-old man presents to the emergency room with epigastric abdominal pain associated with nausea and vomiting. The history reveals that he was binge drinking that evening. He is diagnosed with acute pancreatitis. Which of the following best describes pancreatic function in this patient? a. Pancreatic lipase converts triglycerides into fatty acids and glycerol. b. Phospholipase A2 may be prematurely activated by trypsin. c. Secretin inhibits bicarbonate secretion from the pancreas. d. Serum amylase would be decreased in this patient. e. Serum lipase would be decreased in this patient.

The answer is b

353. A 60-year-old woman presents to her family physician with complaints of paresthesias in her lower legs bilaterally. On physical examination she is found to have a shiny tongue. During the workup, a complete blood count reveals a macrocytic anemia with hypersegmented neutrophils on peripheral smear. She is subsequently diagnosed with pernicious anemia. With respect to cobalamin-intrinsic factor binding in a normal individual, nearly all binding of cobalamin to intrinsic factor occurs in which of the following organs? a. Stomach b. Duodenum c. Jejunum d. Ileum e. Colon

The answer is b

356. A 53-year-old man presents with a chronic cough. The history and physical findings rule out postnasal drip, asthma, and other pulmonary disease. Upon questioning, the patient also reports substernal burning pain that is most pronounced after ingestion of coffee, chocolate, french fries, and alcohol. Which of the following is the most likely cause of the symptoms in this patient? a. Decreased esophageal motility b. Decreased LES tone c. Decreased upper esophageal sphincter tone d. Delayed gastric emptying e. Hiatal hernia

The answer is b

357. A new mother calls the pediatrician because she is concerned that her infant defecates after every meal. Which of the following is the cause of normal bowel movements in newborns? a. Defecation reflex b. Gastrocolic reflex c. Gastroileal reflex d. Intestinointestinal reflex e. Peristaltic rushes

The answer is b

371. A 16-year-old girl presents for her annual high school athletic physical. She states that she seems more tired than usual, she has been having muscle cramps in her calves, and her legs get very weak and sore after running and playing soccer. Her blood pressure is 160/100 mm Hg, and her ECG shows a prolonged QT interval and the presence of a U wave. Blood analysis shows hypokalemia, metabolic alkalosis, and decreases in plasma renin activity and aldosterone concentration. Her clinical condition is reversed after she is placed on the diuretic amiloride. Based on this finding, which of the following renal transport processes is the major defect causing her metabolic disorder? a. Greater than normal sodium reabsorption by the proximal tubules b. Greater than normal sodium reabsorption by the cortical collecting ducts c. Inability of the distal nephron to secrete hydrogen d. Inability of the distal nephron to secrete potassium ion e. Inability to concentrate urine

The answer is b

383. A 46-year-old man presents to his physician with a 12-week history of frontal headaches. CT of the brain shows a mass in the posterior pituitary, and the posterior pituitary "bright spot" is absent on MRI. The patient also complains of increased thirst and waking up frequently during the night. Which of the following best describes his urine? a. A higher-than-normal flow of hypertonic urine b. A higher-than-normal flow of hypotonic urine c. A lower-than-normal flow of hypertonic urine d. A lower-than-normal flow of hypotonic urine e. A normal flow of hypertonic urine

The answer is b

391. An 18-year-old man presents with muscle weakness, cramps, and tetany. Blood pressure is normal and no edema is present. Laboratory analysis reveals hypokalemic alkalosis, hyperaldosteronism, and high plasma renin activity, diagnostic of Bartter's syndrome. Which of the following statements best describes the action or secretion of renin? a. It converts angiotensin I to angiotensin II. b. It converts angiotensinogen to angiotensin I. c. It is secreted by cells of the proximal tubule. d. Its secretion is stimulated by increased mean renal arterial pressure. e. Its secretion leads to loss of sodium and water from plasma.

The answer is b

396. A 14-year-old girl with polycystic kidney disease has a decrease in both GFR and RBF. The nephrologist wants to administer a drug that will increase both GFR and RBF. Both GFR and RBF would increase if which of the following occurred? a. The efferent and afferent arterioles are both constricted. b. The efferent and afferent arterioles are both dilated. c. Only the afferent arteriole is constricted. d. Only the efferent arteriole is constricted. e. The afferent arteriole is constricted and the efferent arteriole is dilated.

The answer is b

398. A 23-year-old woman presents with burning epigastric pain. A careful history reveals that the burning is exacerbated by fasting and improved with meals. The woman is prescribed the H2-receptor antagonist, cimetidine, for suspected peptic ulcer disease. Cimetidine may also have an adverse effect on proximal tubular function. Which of the following substances is more concentrated at the end of the proximal tubule than at the beginning of the proximal tubule? a. Bicarbonate b. Creatinine c. Glucose d. Phosphate e. Sodium

The answer is b

413. A patient has suffered from persistent diarrhea lasting for 7 days. Which of the following would be decreased in this patient? a. Anion gap b. Filtered load of bicarbonate c. H+ secretion by the distal nephron d. Production of ammonia by the proximal tubule e. Production of new bicarbonate by the distal nephron

The answer is b

415. A 54-year-old woman presents with profound fatigue, ankle edema, and paroxysmal nocturnal dyspnea. Filtration fraction may be increased in patients with heart failure due to an increase in which of the following? a. Afferent arteriolar resistance b. Efferent arteriolar resistance c. Hydrostatic pressure within Bowman capsule d. Plasma oncotic pressure e. Renal blood flow

The answer is b

417. A patient with atherosclerosis shows signs of chronic renal failure attributed to poor renal perfusion and ischemic necrosis of the nephrons. Which of the following endogenous substances causes RBF to decrease? a. Acetylcholine b. Angiotensin II c. ANP d. Dopamine e. Nitric oxide

The answer is b

418. A 19-year-old man presents for his annual football physical examination. He is asymptomatic but urinalysis reveals macroscopic hematuria. Microscopic examination is positive for deformed erythrocytes and RBC casts. Where in the renal-urinary system is the most likely origin of the blood in his urine? a. Bowman capsule b. Glomerulus c. Peritubular capillaries d. Renal artery e. Urinary bladder

The answer is b

422. A patient with renal failure presents to the clinic with increasing fatigue for the past month. Based on a thorough history and physical, as well as diagnostic testing, it is determined that the symptoms are caused by the loss of a hormone produced by the kidney. Which of the following is the most likely diagnosis? a. Acidosis b. Anemia c. Edema d. Hypertension e. Uremia

The answer is b

429. Renal and pulmonary biopsies in a 35-year-old woman with Wegener's granulomatosis demonstrate glomerulonephritis and a granulomatous vasculitis in the lungs. In adults, which of the following is greater in the pulmonary circulation compared to the renal circulation? a. Arterial pressure b. Blood flow c. Capillary hydrostatic pressure d. Capillary oncotic pressure e. Vascular resistance

The answer is b

438. A 32-year-old man taking chlorpromazine for his schizophrenia presents with diminished libido and decreased beard growth. His blood prolactin level of 75 µg/L confirms the diagnosis of hyperprolactinemia. Which of the following is true regarding prolactin? a. Prolactin causes milk ejection during suckling. b. Prolactin inhibits GnRH secretion by the hypothalamus. c. Prolactin inhibits gonadotropin secretion by the pituitary gland. d. Prolactin inhibits growth of breast tissue. e. Serum prolactin levels are much higher in women than in men.

The answer is b

441. A 26-year-old man with Klinefelter's syndrome has seminiferous tubule dysgenesis. Which of the following is a function of Sertoli cells in the seminiferous tubules? a. Expression of surface LH receptors b. Maintenance of the blood-testis barrier c. Secretion of FSH into the tubular lumen d. Secretion of testosterone into the tubular lumen e. Synthesis of estrogen after puberty

The answer is b

452. A 55-year-old woman is experiencing the signs and symptoms of menopause. Her gynecologist discusses with her the possibility of hormone replacement therapy (HRT). Which of the following are effects of postmenopausal HRT? a. Increases the risk of osteoporosis b. Reduces the incidence of hot flashes c. Reduces the risk of breast cancer d. Reduces the risk of coronary artery disease and stroke e. Returns the menstrual cycle pattern to normal

The answer is b

465. A 28-year-old woman presents with complaints of vision changes, frequent pressure-like headaches, polyuria, and polydipsia. An MRI of the brain showed a tumor at the posterior pituitary stalk. Which hormone abnormality would be expected? a. Decreased ACTH leading to secondary adrenal insufficiency b. Decreased ADH leading to diabetes insipidus c. Decreased luteinizing hormone (LH) leading to irregular ovulation d. Decreased α-melanocyte-stimulating hormone leading to changes in skin pigmentation e. Decreased TSH leading to hypothyroidism

The answer is b

469. A 24-year-old pregnant woman and her 3-year-old child are seen in a medical mission clinic in Sudan. The child is short in stature, has a potbelly and enlarged protruding tongue, and is developmentally delayed. Iodine is prescribed for mother and child, with the hope of preventing mental retardation in the developing fetus. Iodides are stored in the thyroid follicles mainly in the form of which of the following? a. Monoiodotyrosine b. Thyroglobulin c. Thyroid peroxidase d. Thyroxine e. Triiodothyronine

The answer is b

475. A patient with hyperkalemic renal failure is given an infusion of glucose and insulin. The actions of insulin include which of the following? a. Converting glycogen to glucose b. Enhancing potassium entry into cells c. Increasing plasma amino acid concentration d. Reducing urine formation e. Stimulating gluconeogenesis

The answer is b

481. A 29-year-old man complains of weight gain, decreased energy, dry skin, and brittle hair for the past 6 months. He was diagnosed with hypothyroidism and started on synthetic thyroid hormone. A decrease in which of the following laboratory values would be expected as result of starting treatment? a. Free T4 b. Plasma cholesterol c. Plasma iron d. TSH e. Vitamin A

The answer is b

483. A 20-year-old man with diabetes forgets to take his insulin prior to the start of the National Collegiate Athletic Association (NCAA) swimming championships. Insulin-independent glucose uptake occurs in which of the following sites? a. Adipose tissue b. Brain c. Cardiac muscle d. Skeletal muscle e. Uterus

The answer is b

486. A 13-year-old boy presents for short stature. He was growing appropriately; however, 2 years ago, his primary pediatrician noticed he dropped two percentiles on his growth chart. The patient reports he is much smaller than his friends and has not noticed any pubertal changes such as enlargement of testes or development of axillary or pubic hair. What laboratory test abnormality would you expect to see? a. Decreased ACTH b. Decreased IGF-I c. Increased gonadotropins d. Increased growth hormone e. Increased thyroxine

The answer is b

488. A 59-year-old man is brought to his physician's office by his wife. She reports that he has been weak, nauseated, and urinates frequently. She has also noticed a fruity odor on her husband's breath. A urine sample is strongly positive for ketones and the finger-stick glucose is high, leading to a presumptive diagnosis of diabetes. As a result of insulin deficiency, which of the following will most likely occur? a. Decreased fatty acid release from adipose tissue b. Decreased intracellular α-glycerophosphate in liver and fat cells c. Enhanced glucose uptake and use except by brain tissue d. Increased cellular uptake of glucose e. Indirect depression of glucose utilization due to excess fatty acids in the blood

The answer is b

494. Radiation treatment for a pituitary tumor in an 8-year-old boy results in complete loss of pituitary function. As a result, the child is likely to experience which of the following symptoms? a. Accelerated growth spurts b. Hyporeflexia c. Hyperactivity d. Increased responsiveness to stress e. Sexual precocity

The answer is b

500. A 29-year-old woman presents with paroxysmal episodes of headaches, anxiety, and palpitations. The physician suspects an anxiety disorder, but orders laboratory studies to rule out underlying disease. The laboratory findings of hypercalcemia and elevated urinary catecholamines suggest the possibility of MEN II. Which of the following is the hallmark of pheochromocytoma? a. Dry skin b. Hypertension c. Hypoglycemia d. Lethargy e. Weight gain

The answer is b

60. A 27-year-old African American man who is HIV+ presents with fever, waxing and waning mental status, petechiae, and hematuria. Blood analysis shows thrombocytopenia with normal prothrombin time (PT) and activated partial thromboplastin time (aPTT). Schistocytes are seen on the blood smear. Which of the following is the most likely basis for the pathogenesis of these findings? a. Abnormal sequestration of platelets in the spleen b. Decreased activity of the plasma metalloproteinase ADAMTS13 c. Deficiency of von Willebrand factor (vWF) d. Drug-induced suppression of platelet production e. Sickle cell anemia comorbidity

The answer is b

88. A 78-year-old man is evaluated by a physiatrist after a stroke. The patient is observed to suffer from dysmetria, ataxia, and an intention tremor. These neurological signs are most likely related to a lesion within which of the following regions of the brain? a. Basal ganglia b. Cerebellum c. Cortical motor strip d. Eighth cranial nerve e. Medulla

The answer is b

91. A 64-year-old female patient is referred to a neurologist because her sister and brother both suffered recent strokes. She is diagnosed with an antiphospholipid antibody syndrome, and placed on warfarin. Despite the anticoagulation therapy, she develops a thrombotic cerebral infarct, which leads to spasticity of her left wrist, elbow, and knee. The infarction most likely affected which of the following? a. Ia afferent fibers b. Corticoreticular fibers c. Corticospinal fibers d. Reticulospinal fibers e. Vestibulospinal fibers

The answer is b

92. A 27-year-old patient with a chief complaint of mild vertigo of 3-month duration is seen by a neurologist. Examination reveals a positional (horizontal and vertical) nystagmus that is bidirectional. The patient reports the absence of tinnitus. Which of the following is the most likely etiology of the vertigo? a. Labyrinthitis b. Lesion of the flocculonodular lobe of the cerebellum c. Lesion of the spinocerebellum d. Ménière syndrome e. Psychogenic

The answer is b

110. The morning after a rock concert, a 20-year-old college student notices difficulty hearing his professor during lecture. The physician at the student health center suspects possible damage to his hair cells by the loud music. Depolarization of the hair cells in the cochlea is caused primarily by the flow of which of the following? a. Ca 2+ into the hair cell b. Cl − out of the hair cell c. K+ into the hair cell d. K+ out of the hair cell e. Na+ into the hair cell

The answer is c

113. A 68-year-old man with avitaminosis A presents with a chief complaint of night blindness (nyctalopia). Which of the following reactions in the retinal rods is caused directly by the absorption of light energy? a. Decomposition of scotopsin b. Dissociation of scotopsin and metarhodopsin c. Transformation of 11-cis retinal to all-trans retinal d. Transformation of metarhodopsin to lumirhodopsin e. Transformation of vitamin A to retinene

The answer is c

117. A 59-year-old woman with bilateral glaucoma is treated with drops of the parasympathetic agent pilocarpine. Cholinergic stimulation of the pupil causes which of the following? a. Absence of the pupillary response to light b. Inequality of pupil size c. Pupillary constriction (miosis) d. Pupillary dilation (mydriasis) e. Tonic pupil (slow redilation after exposure to light)

The answer is c

118. A 20-year-old woman complains of altered taste following extraction of her wisdom teeth. Which of the following is the most likely cause of the dysgeusia? a. A disturbed salivary milieu b. Damage to the central gustatory pathways c. Damage to the gustatory afferent nerves d. Impaired transport of the tastant to the receptor cells in the taste bud e. Injury to the taste receptor cells

The answer is c

120. A Jewish couple present at the pediatrician's office with concerns about their 6-month-old child. They report that the infant was perfectly fine for the first few months of life, but lately he is very easily startled by noise, he has difficulty swallowing, he can no longer hold his head up, and he had a seizure earlier in the day. On physical examination, the doctor also notes "cherry-red" spots in the child's eyes. These symptoms are due to the accumulation of harmful quantities of which of the following substances in the brain? a. Beta-amyloid b. Beta-hexosaminidase A c. Ganglioside GM2 d. Sphingomyelin e. Tau protein

The answer is c

124. Three weeks following a gastrointestinal (GI) infection with Campylobacter jejuni, a 60-year-old man develops weakness and tingling in his legs. Over the next few days, his legs and face become paralyzed, and he is hospitalized for Guillain-Barré syndrome (GBS). Which of the following is the most likely underlying cause of his motor paralysis? a. Antibodies against nerve growth factor b. Antibodies against oligodendrogliocytes c. Demyelination of Aβ fibers d. Demyelination of B fibers e. Demyelination of C fibers

The answer is c

126. A 26-year-old woman presents with unilateral facial weakness. She states that whenever she tries to close her eyes, the upper eyelid on the affected side rolls upward. Electromyography on the affected side shows evidence of axonal degeneration. Which of the following characteristics of an axon is most dependent on its diameter? a. The magnitude of its resting potential b. The duration of its refractory period c. The conduction velocity of its action potential d. The overshoot of its action potential e. The activity of its sodium-potassium pump

The answer is c

131. A 37-year-old woman presents with severe migraine headaches that are accompanied by hemiparalysis. Genetic analysis confirms the suspicion of an inherited channelopathy. The membrane potential will depolarize by the greatest amount if the membrane permeability increases for which of the following ions? a. Chloride b. Potassium c. Sodium d. Chloride and potassium e. Sodium and potassium

The answer is c

135. A 58-year-old man with a history of hypertension and renal disease presents at his physician's office with a complaint of headaches. His blood pressure is 190/115 mm Hg and laboratory results show an elevated plasma renin activity with hypernatremia. Which of the following best describes the sodium gradient across the nerve cell membrane? a. It is a result of the Donnan equilibrium b. It is maintained by a Na+/Ca2+ exchanger c. It is significantly changed during an action potential d. It is the primary determinant of the resting membrane potential e. It is used as a source of energy for the transport of other ions

The answer is c

139. An 85-year-old man is brought to his doctor by his daughter. She reports that he has memory loss, is often confused, and has been having increasing difficulty with routine activities that he used to do on his own, such as paying bills and going grocery shopping. She wonders if this is just because of old age or a more serious problem. Which of the following would provide the definitive diagnosis of Alzheimer disease? a. Cerebral cortical atrophy on CT or MRI b. Improved symptoms with cholinesterase inhibitors c. Neuritic plaques containing A-beta (Aβ) amyloid bodies d. Nonspecific slowing of the EEG e. Presence of an apolipoprotein ε4 allele on chromosome 19

The answer is c

142. A 35-year-old woman having an anxiety attack collapses. The emergency medical technician who arrives on the scene notes that she is hyperventilating and has facial and carpopedal spasms. What causes increased excitability of nerves and muscle membranes that can lead to continuous contraction of skeletal muscle fibers? a. Activation of sodium channels at more negative membrane potentials b. Decreased release of inhibitory neurotransmitter from nerve terminals c. Depolarization of the nerve and muscle membranes d. Increased magnitude of the action potentials invading nerve terminals e. Spontaneous release of calcium from the sarcoplasmic reticulum (SR)

The answer is c

143. A 32-year-old woman undergoing an appendectomy develops malignant hyperthermia following halothane anesthesia. What changes in skeletal muscle cause body temperature to increase in this condition? a. An increase in the refractory period of the α-motoneurons b. Inability of skeletal muscle cells to repolarize c. Excess calcium release from the SR during muscle contraction d. Production of endogenous muscle pyrogens e. Rapid repetitive firing of the presynaptic terminals of α-motoneurons

The answer is c

146. An 8-year-old girl experiences two to three episodes of proximal muscle weakness a day. The attacks are precipitated by rest following exercise and last 1 to 2 hours. Serum potassium concentration during the attack of weakness is normal. Which of the following is most likely to cause the muscle weakness in periodic hyperkalemic paralysis? a. Decreased potassium conductance in muscle cells b. Hyperpolarization of muscle cells c. Inactivation of sodium channels in muscle cells d. Increased duration of action potentials produced by α-motoneurons e. Increased release of neurotransmitters from α-motoneurons

The answer is c

146. An 8-year-old girl experiences two to three episodes of proximal muscle weakness a day. The attacks are precipitated by rest following exercise and last 1 to 2 hours. Serum potassium concentration during the attack of weakness is normal. Which of the following is most likely to cause the muscle weakness in periodic hyperkalemic paralysis? a. Decreased potassium conductance in muscle cells b. Hyperpolarization of muscle cells c. Inactivation of sodium channels in muscle cells d. Increased duration of action potentials produced by α-motoneurons e. Increased release of neurotransmitters from α-motoneurons

The answer is c

150. A 20-year-old man presents with a 6-month history of lower back pain of insidious onset, as well as morning low back stiffness that lasts several hours. He reports that the pain improves with exercise but not with rest, and often wakens him at night; when it is relieved, he gets up and walks around. A pelvic MRI reveals sacroiliitis with edema in the juxtaarticular bone marrow, synovium, and joint capsule and interosseous ligaments. Which of the following best describes ankylosing spondylitis? a. Descending progression of endochondral ossification and cartilaginous erosion in the spine. b. It affects women more than men. c. Its occurrence is correlated with the histocompatibility antigen, HLA-B27. d. Peak onset is usually between ages 50 and 60 years. e. Symptoms worsen with the use of tumor necrosis factor-α (TNF-α) inhibitors.

The answer is c

158. A 26-year-old medical student walks briskly down the hallway to grand rounds while eating a hamburger and fries on the way. What is the major difference in the contractile responses occurring in his smooth versus skeletal muscles? a. The mechanism of force generation b. The nature of the contractile proteins c. The role of calcium in initiating contraction d. The source of activator calcium e. The source of energy used during contraction

The answer is c

161. A group of third-year medical students accompanied a medical mission team to Peru, South America. After arriving at the airport in Bolivia, they hiked to a remote mountain village in the Andes at an elevation of 18,000 ft. With a barometric pressure of 380 mm Hg at this altitude, what would be the resulting PO2 of the dry inspired air? a. 160 mm Hg b. 100 mm Hg c. 80 mm Hg d. 70 mm Hg e. 38 mm Hg

The answer is c

162. A 28-year-old man is admitted to the emergency department with multiple fractures suffered in a car accident. Arterial blood gases are ordered while the patient is breathing room air. After the first-year resident obtains an arterial blood sample from the patient, the glass plunger slides back, drawing an air bubble into the syringe before it is handed to the blood gas technician for analysis. How does exposure to room air affect the measured values of PO2 and PCO2 in arterial blood? a. The measured values of both PaO2 and PaCO2 will be higher than the patient's actual values. b. The measured values of both PaO2 and PaCO2 will be lower than the patient's actual values. c. The measured PaO2 will be higher and the measured PaCO2 will be lower than the patient's actual blood gas values. d. The measured PaO2 will be lower and the measured PaCO2 will be higher than the patient's actual blood gas values. e. The measured values of PaO2 and PaCO2 will accurately reflect the actual values.

The answer is c

165. A 150-lb patient scheduled for abdominal surgery is sent for preoperative evaluation and testing. His chest x-ray is normal, and pulmonary function results on room air show the following: Tidal volume = 600 mL Respiratory rate = 12/min Vital capacity = 5000 mL PaO2 = 90 mm Hg PaCO2 = 40 mm Hg PECO2 = 28 mm Hg The volume of the patient's physiological dead space, determined by applying the Bohr equation, equals which of the following? a. 0.3 mL b. 150 mL c. 180 mL d. 420 mL e. 7200 mL

The answer is c

175. A newborn of 28 weeks of gestation develops respiratory distress syndrome. Mechanical ventilation on 100% O2 with 10 cm H2O of positive end-expiratory pressure (PEEP) does not provide sufficient oxygenation. After porcine surfactant is instilled via a fiberoptic bronchoscope, the PaCO2, fraction of inspired oxygen (FIO2), and shunting improve impressively. The improvements in respiratory function occurred because surfactant increased which of the following? a. Alveolar surface tension b. Bronchiolar smooth muscle tone c. Lung compliance d. The pressure gradient needed to inflate the alveoli e. The work of breathing

The answer is c

181. A 67-year-old man who is a candidate for cardiac transplantation undergoes cardiac catheterization to assess his hemodynamic status. Findings include: Pulmonary artery pressure (PAP) = 35 mm Hg Cardiac output = 4 L/min Left atrial pressure (LAP) = 15 mm Hg Right atrial pressure = 10 mm Hg Which of the following values is his PVR? a. 0.16 L/min/mm Hg b. 0.2 L/min/mm Hg c. 5 mm Hg/L/min d. 6.25 mm Hg/L/min

The answer is c

184. Noninvasive color Doppler ultrasound studies are ordered on a term infant and a preterm infant of 28 weeks gestation. Which of the following is likely to have a lower value in the preterm infant compared with the term infant? a. Blood flow from the pulmonary artery through the ductus arteriosus b. Pulmonary artery pressure c. Pulmonary blood flow d. Pulmonary capillary hydrostatic pressure e. Pulmonary vascular resistance

The answer is c

185. A 62-year-old man with congestive heart failure (CHF) develops increasing shortness of breath in the recumbent position. A chest x-ray reveals cardiomegaly, horizontal lines perpendicular to the lateral lung surface indicative of increased opacity in the pulmonary septa, and lung consolidation. Pulmonary edema in CHF is promoted by which of the following? a. Decreased pulmonary capillary permeability b. Decreased pulmonary interstitial oncotic pressure c. Increased pulmonary capillary hydrostatic pressure d. Increased pulmonary capillary oncotic pressure e. Increased pulmonary interstitial hydrostatic pressure

The answer is c

186. A 76-year-old patient with emphysema presents for his annual pulmonary function testing to assess the progression of his disease. As a result of alveolar septal departitioning in emphysema, there is a decrease in which of the following? a. Airway resistance b. Alveolar dead space c. Diffusing capacity d. Lung compliance e. Total lung capacity

The answer is c

195. A 56-year-old man presents to the emergency department with severe abdominal pain and a temperature of 103°F. The patient is in severe respiratory distress. Moderate amounts of pulmonary edema fluid are aspirated during suctioning. The patient is placed on a ventilator with an FIO2 of 0.5 and an arterial blood gas sample reveals a PO2 of 160 mm Hg and a PCO2 of 40 mm Hg. His alveolar oxygen tension, at a barometric pressure of 747 mm Hg and a respiratory exchange ratio (R) of 0.8, is approximately what? a. 100 mm Hg b. 200 mm Hg c. 300 mm Hg d. 400 mm Hg e. 500 mm Hg

The answer is c

196. A 68-year-old man who has COPD presents to his pulmonologist with fatigue, dyspnea at rest, and peripheral edema. His blood gases on room air are PaO2 = 60 mm Hg, PaCO2 = 60 mm Hg, and pH = 7.36. His alveolar-arterial (A-a) O2 gradient, at a barometric pressure of 760 mm Hg and a respiratory exchange ratio (R) of 0.8, is approximately what? a. 5 mm Hg b. 10 mm Hg c. 15 mm Hg d. 20 mm Hg e. 25 mm Hg

The answer is c

203. A 24-year-old presents with a chief complaint of fatigue and daytime somnolence. His wife has noticed that he stops breathing for periods of 30 to 60 seconds while he is sleeping and that this happens many times throughout the night. His physician orders pulmonary function testing including ventilatory response curves and polysomnography. The tests confirm apneic episodes during sleep. During a ventilatory responsiveness test, his alveolar ventilation increased as predicted in response to breathing 5% CO2, but his ventilatory response to breathing 16% O2 was depressed. Which of the following conditions are consistent with these findings? a. Central hypoventilation syndrome (Ondine curse) b. Decreased central chemoreceptor sensitivity c. Decreased peripheral chemoreceptor sensitivity d. Obstructive sleep apnea e. Spinal cord injury affecting the fourth cervical vertebra

The answer is c

206. A 48-year-old coal miner complains of shortness of breath and a productive cough. He has smoked one to two packs of cigarettes per day since he was 16 years old. Pulmonary function studies are ordered, including an esophageal balloon study to measure intrapleural pressures. Normally, intrapleural pressure is negative throughout a tidal inspiration and expiration because of which of the following? a. The lungs have the tendency to recoil outward throughout a tidal breath. b. The chest wall has the tendency to recoil inward throughout a tidal breath. c. The lungs and chest wall recoil away from each other throughout a tidal breath. d. The lungs and chest wall recoil in the same direction throughout a tidal breath. e. A small volume of air leaves the pleural space during a tidal breath.

The answer is c

210. A 37-year-old woman is admitted to the hospital with severe kyphoscoliosis and respiratory muscle weakness. Which of the following physiological variables is most likely decreased in this patient? a. Airway resistance b. Alveolar surface tension c. Arterial carbon dioxide tension d. Chest wall compliance e. FEV1/FVC

The answer is c

214. A 29-year-old man with AIDS presents with a painful, red, swollen area on top of his shin, which is warm to the touch. He has a fever, tachypnea, and tachycardia, and is hospitalized and started on IV antibiotics. His condition progresses rapidly to septicemia and septic shock. He is transported to the ICU, intubated, and started on mechanical ventilation. A Swan-Ganz catheter is inserted to monitor pulmonary hemodynamics and lung fluid balance. Which of the following conditions will cause a decrease in PVR? a. Alveolar hypoxia b. Decreased pH in the pulmonary artery c. Increased cardiac output d. Inflation of the lungs to total lung capacitye. Sympathetic stimulation of the pulmonary vessels

The answer is c

219. A 36-year-old man visits his doctor because his wife has long complained of his snoring, but recently observed that his breathing stops for a couple of minutes at a time while he is sleeping. He undergoes polysomnography and ventilatory response testing to ascertain the extent and cause of his sleep apnea. The activity of the central chemoreceptors is stimulated by which of the following? a. A decrease in the metabolic rate of the surrounding brain tissue b. A decrease in the PO2 of blood flowing through the brain c. An increase in the PCO2 of blood flowing through the brain d. An increase in the pH of the CSF e. Hypoxemia, hypercapnia, and metabolic acidosis

The answer is c

23. A patient with congestive heart failure presents with jugular venous distention, ascites, and peripheral edema. Blood work shows elevated levels of plasma ANP. ANP increases Na+ excretion by which of the following mechanisms? a. Contracting afferent arterioles b. Decreasing GFR c. Decreasing sodium reabsorption by the inner medullary collecting duct d. Increasing permeability of the apical membrane of the collecting duct epithelial cells e. Increasing sodium reabsorption by the proximal tubules

The answer is c

235. The spouse of a 58-year-old man calls 911 because her husband complains of chest pain radiating down his left arm. He is transported to the emergency department, where an ECG and cardiac enzymes indicate a recent myocardial infarction. The man undergoes cardiac catheterization, including coronary angiography and hemodynamic recordings throughout the cardiac cycle. No valvular defects were present. During ventricular ejection, the pressure difference smallest in magnitude is between which of the following? a. Aorta and capillaries b. Left atrium and left ventricle c. Left ventricle and aorta d. Pulmonary artery and left atrium e. Right atrium and right ventricle

The answer is c

241. A patient undergoes cardiac transplantation for severe idiopathic cardiomyopathy. Upon release from the hospital, the patient is referred to a cardiac rehabilitation program. The exercise physiologist starts the patient on a walking regimen. During exercise in cardiac transplant patients, cardiac output increases primarily due to an increase in which of the following? a. Arterial blood pressure b. Heart rate c. Stroke volume d. TPR e. Venous compliance

The answer is c

246. A 72-year-old man is hospitalized with a history of respiratory distress, fever, and fatigue. An ECG reveals ST-segment and T-wave abnormalities and echocardiography shows an ejection fraction of 30%. Over the next several days, significant peripheral edema develops. The edema is most likely caused by which of the following? a. Decreased capillary permeability b. Decreased arterial pressure c. Increased central venous pressure d. Increased lymphatic flow e. Increased plasma protein concentration

The answer is c

250. A 68-year-old man presents for his quarterly cardiology examination. Upon auscultation, a third heart sound is heard. Of the following conditions, which is the most likely cause of an S3? a. Aortic regurgitation b. Aortic stenosis c. Heart failure d. Right bundle branch block e. Tricuspid regurgitation

The answer is c

267. A 36-year-old athlete becomes alarmed when he notices a series of heart palpitations several hours after he exercises. After examining the patient's ECG, the physician notes a sinus rhythm with occasional unifocal premature ventricular complexes (PVCs). Which of the following may predispose an athlete to the occurrence of PVCs? a. An accessory bundle of Kent b. Atrial fibrillation c. Bradycardia d. Inverted P waves e. Sinus tachycardia

The answer is c

270. A 47-year-old man is brought to the emergency department because he had chest pain, was short of breath, and fainted at the gym during his daily workout. A prominent systolic ejection click and crescendodecrescendo systolic murmur is heard over the right sternal border. Which of the following findings is consistent with the patient's most likely diagnosis? a. Decreased cardiac oxygen consumption b. Decreased left ventricular systolic pressure c. Decreased pulse pressure d. Increased arterial blood pressure e. Increased ejection fraction

The answer is c

277. A 40-year-old woman with metabolic syndrome is prescribed a low-calorie diet and 30 minutes of daily aerobic exercise. Sympathetic stimulation during exercise has which of the following effects on the heart? a. A decrease in the cytosolic concentration of Ca2+ during systole b. A decrease in intracellular concentration of cyclic AMP c. An increase in the activity of the sarcoplasmic reticulum (SR) calcium pump d. An increase in the duration of diastole e. An increase in the duration of systole

The answer is c

279. A 23-year-old collegiate dance squad member adopts a sedentary lifestyle once she starts medical school. After the gross anatomy course, she decides to restore her state of physical fitness by resuming a regular exercise routine. The cardiovascular responses to isotonic exercise include an increase in which of the following? a. Diastolic pressure b. Pulmonary vascular resistance c. Stroke volume d. Systemic vascular resistance e. Venous compliance

The answer is c

291. A 6-day-old baby girl undergoes a routine physical examination. She is found to be tachycardic, and has a wide pulse pressure. A thrill and a continuous murmur with late systolic accentuation at the upper left sternal edge are detected upon auscultation. Which of the following best describes the in utero function of the most likely structure causing the murmur? a. It allows blood to flow from the aorta to the pulmonary artery. b. It delivers oxygenated blood from the placenta to the left ventricle. c. It diverts oxygenated blood away from the lungs to the aorta. d. It is a high-resistance conduit, which helps to maintain normal fetal blood pressure. e. It is located in the septum between the left and right atrium.

The answer is c

296. A 28-year-old woman gave birth without complications 48 hours ago to a term 8-lb 12-oz boy. Which of the following best describes the functional closure of the ductus arteriosus? a. It causes blood to flow from the aorta into the pulmonary artery. b. It is independent of gestational age. c. It is the final event required for conversion of the transitional circulation in the newborn to the adult circulatory pattern. d. It occurs due to hypoxic pulmonary vasoconstriction. e. It precedes functional closure of the foramen ovale.

The answer is c

298. A 59-year-old man with an ejection fraction of 15%, who is being treated with medications for his heart failure, is asked whether he would like to participate in a trial for an experimental drug. The drug being tested is designed to decrease the expression of phospholamban on ventricular muscle cells. Which of the following would be increased by decreasing phospholamban? a. Activity of the L-type calcium channels b. Activity of the sodium-potassium pump c. Concentration of calcium within the SR d. Diastolic stiffness of the ventricular muscle cells e. Duration of the ventricular muscle action potential

The answer is c

302. A 64-year-old man was admitted to the hospital with edema and CHF. He was found to have diastolic dysfunction characterized by inadequate filling of the heart during diastole. The decrease in ventricular filling is due to a decrease in ventricular muscle compliance. Which of the following proteins determines the normal stiffness of ventricular muscle? a. Calmodulin b. Myosin light chain kinase c. Titin d. Tropomyosin e. Troponin

The answer is c

325. A newborn with severe diarrhea is found to have an inherited defect in a glucose transporter resulting in glucose/galactose malabsorption, necessitating a glucose and galactose-free diet. Which of the following is the transport protein responsible for entry of glucose into the intestinal enterocyte? a. Glut-2 b. Glut-5 c. SGLT 1 d. SGLT 2 e. SGLT 5

The answer is c

331. A 43-year-old woman presents with chief complaints of bulky and frequent diarrhea and weight loss. She experiences recurrent episodes of abdominal distension terminated by passage of stools. Laboratory data reveals a microcytic anemia, decreased serum calcium, and decreased serum albumin. Her generalized decrease in intestinal absorption can be attributed to which of the following? a. Decreased gastric emptying b. Decreased intestinal motility c. Decreased intestinal surface area d. Increased enterohepatic circulation of bile e. Increased migrating motor complexes

The answer is c

350. A 38-year-old man has dinner one evening at his favorite steakhouse. Several hours later, the chyme reaches the duodenum. After secretion of trypsinogen into the duodenum, the enzyme is converted into its active form, trypsin, by which of the following? a. Alkaline pH b. Chymotrypsin c. Enteropeptidase d. Pancreatic lipase e. Procarboxypeptidase

The answer is c

351. An 18-year-old woman decides to get a tattoo for her birthday. Two months later she presents with a fever, right upper quadrant pain, nausea, vomiting, and jaundice. Which of the following laboratory values would most likely be found in a patient with infectious hepatitis? a. A decrease in both direct and indirect plasma bilirubin b. A decrease in plasma alkaline phosphatase c. An increase in both direct and indirect plasma bilirubin d. An increase in plasma alkaline phosphatase e. An increase in plasma bile acids

The answer is c

355. A 52-year-old woman who has been dieting for several weeks breaks down and eats half a pan of frosted brownies. Insulin secretion following a carbohydrate-rich meal is stimulated by which of the following? a. Cholecystokinin b. Gastrin c. Glucagon-like polypeptide 1 (GLP-1) d. Serotonin e. VIP

The answer is c

365. Twenty years ago, a 65-year-old man underwent vagotomy for his refractory peptic ulcer disease. As a result, which of the following GI motor activities will be affected most? a. Distention-induced intestinal segmentation b. Migrating motor complexes c. Orad stomach accommodation d. Secondary esophageal peristalsis

The answer is c

368. A full-term newborn infant with abdominal distention has not had a bowel movement for 5 days. An x-ray of the abdomen shows a narrowed colon, bowel obstruction, and dilated intestine above the obstruction. A suction rectal biopsy is done at the bedside. The pathology report indicates the absence of ganglion cells and the presence of nonmyelinated nerves in the biopsy segment. What is the underlying cause of the bowel obstruction in this patient? a. Activation of intestinal opioid receptors b. Carcinoid tumor c. Impaired endothelin B receptor function d. Impaired gastrocolic reflex e. Increased activity of splanchnic sympathetic nerves

The answer is c

375. A 28-year-old woman presents to her physician's office with fatigue, malaise, and orthostatic dizziness. When asked what medications she is taking, she stated that she has been taking 800 mg ibuprofen four to six times a day for painful menstrual cramps. Serum creatinine was elevated to 2.1 mg/dL. Which of the following is most likely to produce an increase in GFR in patients with acute renal failure? a. Administration of angiotensin II b. Contraction of glomerular mesangial cells c. Dilation of afferent arterioles d. Increased renin release from the juxtaglomerular apparatus e. Volume depletion

The answer is c

379. An 85-year-old woman presents with a fever and hypovolemic hypotension. To assess her renal function, the filtration fraction is determined using a freely filterable substance that is neither reabsorbed nor secreted. The infusate yields a renal artery concentration of 12 mg/mL and a renal vein concentration of 9 mg/mL. Which of the following is her filtration fraction? a. 0.05 b. 0.15 c. 0.25 d. 0.35 e. 0.45

The answer is c

382. A 68-year-old woman presents with hypertension and oliguria. A CT of the abdomen reveals a hypoplastic left kidney. Based on the following laboratory data, which of the following is her estimated RPF? Renal artery p-aminohippuric acid (PAH) = 6 mg/dL Renal vein PAH = 0.6 mg/dL Urinary PAH = 25 mg/mL Urine flow = 1.5 mL/min Hematocrit = 40% a. 475 mL/min b. 550 mL/minc. 625 mL/min d. 700 mL/min e. 775 mL/min

The answer is c

389. A 92-year-old man presents with dehydration following 4 days of persistent diarrhea. Under this circumstance, hypotonic fluid would be expected in which of the following? a. Glomerular filtrate b. Proximal tubule c. Ascending limb of the loop of Henle d. Cortical collecting tubule e. Distal collecting duct

The answer is c

400. A 17-year-old male presents with fatigue, muscle cramps, and joint pain. Blood analysis reveals hypokalemia, hypomagnesemia, and hypochloremic metabolic alkalosis, and urinalysis reveals decreased urinary chloride and calcium. The clinical findings suggest a loss-of-function mutation of the SLC12A3 gene encoding thethiazide-sensitive sodium-chloride cotransporter (NCCT). Electrically neutral active transport of sodium and chloride occurs in which of the following areas of the nephron? a. Cortical collecting duct b. Descending limb of the loop of Henle c. Distal tubule d. Medullary collecting duct e. Thin ascending limb of the loop of Henle

The answer is c

401. A 36-year-old African American man presents with low renin essential hypertension. Renin release from the juxtaglomerular apparatus is normally inhibited by which of the following? a. Aldosterone b. β-Adrenergic agonists c. Increased pressure within the afferent arterioles d. Prostaglandins e. Stimulation of the macula densa

The answer is c

430. A 65-year-old man presents in the emergency department with a fracture of his right arm after slipping and falling on the ice. He reports that he has had back pain for the past 6 months. Blood results show Hb = 9 g/dL; hematocrit = 30%; BUN = 35 mg/dL; creatinine = 3 mg/dL. Urinalysis shows pH >5.3 and is positive for Bence Jones proteins. The patient is diagnosed with type II (proximal) RTA secondary to multiple myeloma. The transport of H+ into the proximal tubule is primarily associated with which of the following? a. Excretion of ammonium ion b. Excretion of potassium ion c. Reabsorption of bicarbonate ion d. Reabsorption of calcium ion e. Reabsorption of phosphate ion

The answer is c

431. A 57-year-old woman with chronic cardiac failure presented at the University Medical Center to participate in a clinical research study on the genetics of heart failure. Genetic analysis showed an increase in ADH gene expression and associated hypothalamic biosynthesis of the hormone, in addition to increased release of the hormone from the posterior pituitary. In the presence of ADH, the filtrate will be isotonic to plasma in which part of the kidney? a. Ascending limb of the loop of Henle b. Descending limb of the loop of Henle c. Cortical collecting tubule d. Medullary collecting tubule e. Renal pelvis

The answer is c

434. A 28-year-old woman with blood pressures that have ranged from 155-190/70-100 during her last two visits to her family physician is started on lisinopril. When her blood pressure worsened on lisinopril, she was referred to a nephrologist for uncontrolled hypertension. Her blood pressure was 170/95 and a renal ultrasound showed a greater than 50% reduction in vessel diameter. Based on the laboratory data shown below, what is her estimated renal blood flow? Serum inulin = 2 mg/dL Urinary inulin = 10 mg/mL Serum PAH = 6 mg/dL Urinary PAH = 20 mg/mL Urine flow = 1.2 mL/min Hematocrit = 40% Hemoglobin = 13 a. 400 mL/min b. 600 mL/min c. 667 mL/min d. 1000 mL/min e. 1250 mL/min

The answer is c

437. A 19-year-old female presents to her primary care physician with significant weight loss and secondary amenorrhea. She has a high-intensity exercise regimen, is preoccupied with food, and seems to have an irrational fear of gaining weight. Decreased production of which of the following hormones leads to amenorrhea in anorexia nervosa? a. Human chorionic gonadotropin (hCG) b. Estradiol c. Gonadotropin releasing hormone (GnRH) d. Progesterone e. Prolactin

The answer is c

446. A woman presents to her obstetrician with concerns that she has had trouble breast feeding. She reports that her mother-in-law told her that beer would relax her and allow her milk to flow more readily, but it has not helped, even with drinking up to a six pack a day. Which of the following hormones is involved in the ejection of milk from a lactating mammary gland? a. Estrogen b. Growth hormone c. Oxytocin d. Progesterone e. Prolactin

The answer is c

450. A young couple presents with concerns that they have not been able to conceive a child. Physical examination of the 22-year-old husband reveals mild obesity, gynecomastia, and decreased facial and axillary hair. He has male genitalia, but penile length is decreased and the testes are small. Chromosomal analysis reveals the XXY pattern of Klinefelter's syndrome. Which of the following is the principal androgen responsible for transforming undifferentiated external genitalia in the fetus into male external genitalia? a. Androstenedione b. Androsterone c. Dihydrotestosterone d. Müllerian-inhibiting substance (MIS) e. Testosterone

The answer is c

451. A 12-year-old girl presents to her pediatrician's office because she has not yet begun her menstrual periods and she lacks breast development. After evaluation, she is found to have Turner's syndrome. Which of the following best describes a patient with Turner's syndrome? a. It is not associated with hypothyroidism. b. It is not associated with renal abnormalities. c. Ovarian dysgenesis (streak ovary) is characteristic. d. Tall stature is common. e. The most common karyotype is 45,X/46,XX mosaicism.

The answer is c

466. A 36-week pregnant mother has a decrease in urinary estriol excretion, indicating a decline in fetal adrenal cortical activity. Which of the following is the principal steroid secreted by the fetal adrenal cortex? a. Cortisol b. Corticosterone c. Dehydroepiandrosterone d. Progesterone e. Pregnenolone

The answer is c

468. A 22-year-old woman presents with a recurrent vaginal candidiasis that is refractory to nystatin treatment. Diabetes screening shows elevated fasting blood glucose, and the patient is started on 25 U of insulin per day. Which aspect of glucose transport is enhanced by insulin? a. Transport across the tubular epithelium of the kidney b. Transport against a concentration gradient c. Transport into adipocytes d. Transport into the brain e. Transport through the intestinal mucosa

The answer is c

473. A 20-year-old man presents with increasing daytime somnolence. A 24-hour sleep study showing a sudden onset of rapid eye movement (REM) sleep without previous slow-wave sleep confirms a diagnosis of narcolepsy. REM sleep decreases the secretion of growth hormone. The physiological secretion of growth hormone is increased by which of the following? a. Free fatty acids b. Growth hormone c. Hypoglycemia d. Hyperglycemia e. Somatostatin

The answer is c

479. An abdominal computed tomography (CT) in a 50-year-old patient with Conn's syndrome (primary hyperaldosteronism) shows multiple small adrenocortical masses. Which of the following clinical findings are most likely present? a. Decreased extracellular fluid volume b. Hyperkalemia c. Hypertension d. Increased concentrating ability of the kidney e. Increased hematocrit

The answer is c

480. A 75-year-old woman with primary hyperparathyroidism presents at her physician's office with dehydration and malaise. Which of the following plasma levels are most likely to be decreased? a. Calcitonin b. Calcium c. Phosphate d. Potassium e. Sodium

The answer is c

485. A multisystem trauma patient develops hyperpyrexia, severe tachycardia, and high-output congestive heart failure with volume depletion, consistent with thyroid storm. Which of the following is the most appropriate treatment for the exaggerated hyperthyroidism? a. Aspirin to treat fever b. β-Adrenergic antagonist therapy to block sympathomimetic symptoms c. Iodine followed by propylthiouracil to block release and synthesis of thyroid hormone d. Oral hydration to correct volume depletion e. T3 administration to induce negative feedback inhibition of T4

The answer is c

493. A 34-year-old patient with chronic asthma is started glucocorticoid therapy. The treatment may result in bone loss because glucocorticoids do which of the following? a. Increase calcium absorption from the gastrointestinal tract b. Increase osteoblast growth c. Inhibit bone formation d. Inhibit bone resorption e. Suppress vitamin D activation

The answer is c

59. A 21-year-old woman presents to her doctor's office indicating that she has been feeling very tired for the past 2 weeks and has recently developed a sore throat and fever. Urine output has been normal. Her temperature is 100.7°F (38.1°C) and blood pressure is 100/70 mm Hg. Lung fields are clear on auscultation, but there are exudates on the pharynx, the posterior cervical nodes are enlarged and tender, and she has splenomegaly. The white blood cell count is 20,000/μL with lymphocytosis and greater than 10% atypical lymphocytes. Rapid antigen testing for gram-negative bacteria is negative and serologic testing shows a positive reaction for heterophile antibody. Based on these findings, which of the following is the most likely diagnosis? a. β-Hemolytic Streptococcus infection b. Hepatitis B c. Infectious mononucleosis d. Influenza e. Toxic shock syndrome

The answer is c

61. A 42-year-old patient is scheduled for surgery that will likely require a transfusion. Because the patient has a rare blood type, an autologous blood transfusion is planned. Prior to surgery, 1500 mL of blood is collected. The collection tubes contain calcium citrate to prevent coagulation. Which of the following is the mechanism for citrate's anticoagulative action? a. Activating plasminogen b. Binding factor XII c. Binding vitamin K d. Blocking thrombin e. Chelating calcium

The answer is c

62. Preoperative evaluation of a 56-year-old man scheduled for knee replacement surgery reveals frequent bruising and a family history of a bleeding disorder, but no bleeding problems himself. The patient is found to have a normal PT, increased aPTT, and a Factor VIII activity level of 40%. These findings are indicative of which of the following conditions? a. Bernard-Soulier syndrome b. Glanzmann thrombasthenia c. Hemophilia A d. Hemophilia B e. von Willebrand disease

The answer is c

63. A 44-year-old woman with a history of excessive menstrual bleeding and menstrual cycles that generally last over 7 days complains of increasing fatigue and cold extremities. Laboratory results reveal a hemoglobin (Hb) concentration of 6 g/dL. In this patient with anemia, which of the following would be reduced? a. Arterial PO2 b. Dissolved oxygen content c. Oxygen extraction d. Percent O2 saturation in the arterial blood e. Total arterial oxygen content

The answer is c

71. A 23-year-old man with a ruddy complexion presents with chief complaints of headache, dizziness, and lethargy. Blood analysis shows erythrocytosis and a P50 of 20 mm Hg. He denies any history of tobacco smoking and is unaware of any other exposure to carbon monoxide or nitrites. Which of the following is a probable cause for these findings? a. α-Thalassemia-2 b. β-Thalassemia major c. High-O2 affinity hemoglobinopathy d. Low-O2 affinity hemoglobinopathy e. Sickle cell trait

The answer is c

75. A 52-year-old man is brought to the emergency department with severe chest pain. Angiography demonstrates a severe coronary occlusion. A thrombolytic agent is administered to reestablish perfusion. Which of the following does the thrombolytic agent activate? a. Heparin b. Kininogen c. Plasminogen d. Prothrombin e. Thrombin

The answer is c

83. A 13-year-old adolescent boy has no movement in his legs after falling out of a tree. Neurological examination shows the absence of both the myotatic (stretch) and reverse myotatic reflexes in the lower extremities. Which of the following is the most important role of the γ-motoneurons? a. Detect the length of resting skeletal muscle b. Generate activity in Ib afferent fibers c. Maintain Ia afferent activity during contraction of muscle d. Prevent muscles from producing too much force e. Stimulate skeletal muscle fibers to contract

The answer is c

97. A 59-year-old woman with an inherited neurodegenerative disease is admitted to the hospital because of agitation and aggression. Three years prior to admission, her irregular, flinging movements had become so severe that she could not walk or assist in her own care. Within which of the following areas of the brain the neuronal degeneration results in this presentation? a. Anterior cerebellum b. Limbic system c. Striatum d. Substantia nigra e. Subthalamus

The answer is c

99. A 27-year-old musician visits an otolaryngologist complaining of ringing in his ear. An audiometry test reveals a high-frequency hearing loss in which the threshold for hearing high-frequency sounds is raised by 1000 times. If a patient is unable to hear high-frequency sounds, the damage to the basilar membrane is closest to which of the following structures? a. Helicotrema b. Modiolus c. Oval window d. Spiral ganglion e. Stria vascularis

The answer is c

101. A 41-year-old man complains to his physician about jet lag whenever he flies long distances to meetings. Melatonin is prescribed as a way to reset his circadian rhythm. The circadian rhythm is controlled by which of the following nuclei? a. Arcuate b. Lateral c. Paraventricular d. Suprachiasmatic e. Ventromedial

The answer is d

134. A 13-year-old boy on the junior high wrestling team experienced attacks of proximal muscle weakness that lasted from 30 minutes to as long as 4 hours following exercise and fasting. The trainer attributed it to the symptoms of fatigue, but his mother recalled having similar symptoms when she was on a diet and exercise regime. Genetic testing revealed an inherited channelopathy. Electrically excitable gates are normally involved in which of the following? a. Depolarization of the end-plate membrane by acetylcholine b. Hyperpolarization of rods by light c. Increase in nerve cell potassium conductance caused by membrane depolarization d. Release of calcium from ventricular muscle sarcoplasmic reticulum e. Transport of glucose into cells by a sodium-dependent, secondary active transport system

The answer is d

138. A 10-year-old girl with type I diabetes develops a neuropathy of sensory neurons with free nerve endings. As a result, quantitative sensory testing will reveal higher-than-normal thresholds for detection of which of the following stimuli? a. Fine touch b. Muscle length c. Pressure d. Temperature e. Vibration

The answer is d

147. A 16-year-old adolescent boy on the track team asks his pediatrician if he can take creatine on a regular basis in order to increase his muscle strength prior to a track meet. Which of the following most likely explains why he wants to take creatine? a. Creatine delays the metabolism of fatty acids. b. Creatine increases muscle glycogen concentration. c. Creatine increases plasma glucose concentration. d. Creatine is converted to phosphorylcreatine. e. Creatine prevents dehydration.

The answer is d

148. An 18-month-old boy presents with delayed dentation, short stature, difficulty and painful walking, and bowing of the legs. The history reveals that he has been breast-fed, but that he has not been receiving daily vitamin D supplementation. A defect in which of the following can explain his findings? a. Blood supply to the haversian canals b. Bone formation by osteoblasts c. Bone resorption by osteoclasts d. Calcification of the bone matrix e. Composition of bone collagen

The answer is d

156. An 87-year-old man presents with acute pain and swelling of the right knee. He is subsequently diagnosed with calcium pyrophosphate dihydrate (CPPD) deposition (pseudogout) after joint aspiration. Which of the following would be expected with CPPD deposition? a. Aspiration reveals weakly negative birefringent crystals. b. Rhomboid crystals are not seen in the aspirate. c. There is decreased production of inorganic pyrophosphate. d. The knee is the most commonly affected joint. e. The temporomandibular joint (TMJ) is commonly involved.

The answer is d

157. A 28-year-old man qualifies to run the Boston marathon. He undertakes an endurance training regimen designed to improve marathon performance. Which of the following properties is greater in Type I compared to Type IIb/x skeletal muscle fibers, thereby promoting distance running success? a. Diameter of the muscle fibers b. Force of contraction c. Glycolytic capacity d. Oxidative capacity e. Speed of contraction

The answer is d

159. A 62-year-old woman presents to her primary care physician with a 2-month history of stiffness and aching in her neck, shoulders, and hips. She is referred to a rheumatologist. After doing a battery of tests, he diagnoses the woman with polymyalgia rheumatica and not fibromyalgia. Which of the following findings is characteristic of polymyalgia rheumatica? a. Absence of response to prednisone b. Antibodies to vascular smooth muscle c. Increased serum creatine kinase (CK) levels d. Increased erythrocyte sedimentation rate (ESR) e. Persistent muscle weakness

The answer is d

169. A 125-lb, 40-year-old woman with a history of nasal polyps and aspirin sensitivity since childhood presents to the emergency department with status asthmaticus and hypercapnic respiratory failure. She requires immediate intubation and is placed on a mechanical ventilator on an FIO2 of 40%, a control rate of 15 breaths per minute, and a tidal volume of 500 mL. Which of the following is her approximate alveolar ventilation? a. 375 mL/min b. 3500 mL/min c. 5250 mL/min d. 5625 mL/min e. 7500 mL/min

The answer is d

172. A 36-year-old man with a history of AIDS and Pneumocystis infection presents to the emergency department with severe respiratory distress. The patient is placed on a ventilator at a rate of 16, tidal volume of 600 mL, and FIO2 of 1.0. An arterial blood sample taken 20 minutes later reveals a PO2 of 350 mm Hg, a PCO2 of 36 mm Hg, and a pH of 7.32. At a barometric pressure of 757 mm Hg, and assuming a normal respiratory exchange ratio (R) of 0.8, the patient's alveolar oxygen tension is approximately which of the following? a. 105 mm Hg b. 355 mm Hg c. 576 mm Hg d. 665 mm Hg e. 712 mm Hg

The answer is d

197. A 45-year-old man presents with severe back pain that he attributes to an injury from operating a jackhammer for his job as a cement worker. An MRI of the spine confirms a herniated disk. The patient reports that he has smoked one to two packs of cigarettes a day for 30 years, so the neurosurgeon requests pulmonary function studies prior to the patient's back surgery. During a forced expiration, the patient generates an intrapleural pressure of 20 mm Hg. The patient's equal pressure point will move closer to the mouth and forced expiratory volume will increase if there is an increase in which of the following? a. Airway resistance b. Airway smooth muscle tone c. Expiratory effort d. Inspired lung volume e. Lung compliance

The answer is d

199. A 5-month-old infant is admitted to the hospital for evaluation because of repeated episodes of sleep apnea. During a ventilatory response test, his ventilation did not increase when PaCO2 was increased, but decreased during hyperoxia. Which of the following is the most likely cause of this infant's apnea? a. Bronchospasm b. Decreased irritant receptor sensitivity c. Diaphragmatic fatigue d. Dysfunctional central chemoreceptors e. Peripheral chemoreceptor hypersensitivity

The answer is d

216. A 56-year-old woman presents to her physician complaining of fatigue, headaches, and dyspnea on exertion. She states that she sometimes gets blue lips and fingers when she tries to exercise. Pulmonary function tests reveal an increase, rather than a decrease, in the diffusing capacity of the lung. Which of the following conditions best accounts for an increase in the diffusing capacity? a. CHF b. COPD c. Fibrotic lung disease d. Polycythemia e. Pulmonary embolism

The answer is d

236. A 55-year-old man reports several episodes of syncope and worsening exercise intolerance over the past year. Of the following, which is the most likely diagnosis? a. Sinus arrhythmia b. First-degree heart block c. Second-degree heart block d. Third-degree heart block e. Multifocal atrial tachycardia

The answer is d

242. A patient complaining of an irregular heartbeat is referred for a cardiac electrophysiological study. Propagation of the action potential through the heart is fastest in which of the following cardiac structures? a. SA node b. Atrial muscle c. Atrioventricular (AV) node d. Purkinje fibers e. Ventricular muscle

The answer is d

249. A 24-year-old woman undergoes an annual physical examination for participation on the varsity track team at her college. While auscultating her heart sounds, the sports medicine physician instructs the woman to take in a deep inspiration. During this maneuver, he detects normal splitting of the second heart sound. Which of the following is the mechanism underlying this finding? a. A decrease in heart rate b. An increased left ventricular stroke volume c. Delayed closing of the aortic valve d. Delayed closing of the pulmonic valve e. Delayed opening of the mitral valve

The answer is d

257. A newborn baby is cyanotic upon delivery. The cyanosis is not relieved by breathing 100% oxygen. A diagnosis of persistent fetal circulation is made based on which of the following? a. Aortic coarctation b. Left ventricular hypertrophy c. Mitral regurgitation d. Pulmonary vasoconstriction and hypertension e. Systemic hypertension

The answer is d

269. Cardiac catheterization is performed on a 39-year-old man who presents with angina. The left ventricular pressure-volume curve shows a decreased stroke volume and ejection fraction. Which of the following mechanisms may compromise stroke volume following myocardial infarction? a. Decreased arterial blood pressure b. Decreased total peripheral resistance c. Increased central venous pressure d. Increased heart rate e. Sympathetic-mediated positive inotropy

The answer is d

271. A patient with an inferior MI develops a stable bradycardia of 50 beats per minute (bpm). The cardiologist orders an ECG to evaluate whether there is sinus node dysfunction or an AV conduction disturbance. The diagnosis of a first-degree heart block is made in which of the following cases? a. Asynchrony of P waves and QRS complexes b. Fixed, prolonged PR interval followed by a nonconducted QRS complex at regular intervals c. Normal PR interval, normal QRS complexes, increased R-R interval d. Prolonged PR interval with every P wave followed by a QRS complex e. The PR interval is progressively prolonged until a QRS complex is dropped

The answer is d

275. A 43-year-old man comes to his physician complaining of exhaustion and shortness of breath. After completing the physical examination, the physician suspects the patient may be suffering from pericardial tamponade. Which of the following observations led to the physician's putative diagnosis? a. Bradycardia b. Expiratory rales c. Hypertension d. Pulsus paradoxus e. Third heart sound

The answer is d

288. A 29-year-old woman presents at the office of her obstetrician/gynecologist with breast tenderness. She reports that her last menstrual period was 6 weeks ago. An assay for human chorionic gonadotropin in her urine is positive. During pregnancy, which of the following is true of the maternal and fetal circulations? a. Fetal CO2 that diffuses across the placenta is removed by the maternal uterine arteries. b. The oxygen dissociation curve for fetal hemoglobin is shifted to the right of that of maternal hemoglobin A. c. The highest partial pressure of oxygen in the fetus is in the umbilical arteries. d. The majority of the fetal cardiac output goes to the placenta. e. Uterine blood flow doubles.

The answer is d

300. A 62-year-old man with a history of diabetes mellitus and hypertension arrives in the emergency room with substernal chest pain for the last hour. He is given intravenous nitroglycerin to help reduce the pain. Which of the following would be expected with the use of this drug? a. Arterial blood pressure is increased. b. Coronary blood flow is decreased. c. Left ventricular wall stress is increased. d. Myocardial oxygen demand is decreased. e. Venous return to the heart is increased.

The answer is d

303. A 22-year-old man with no history of congenital heart disease has a normal physical examination prior to entering the military. Which of the following characteristics is most similar in the systemic and pulmonary circulations of this patient? a. Afterload b. Blood volume c. Peak systolic pressure d. Preload e. Stroke work

The answer is d

313. An 18-year-old man with pernicious anemia lacks intrinsic factor, which is necessary for the absorption of cyanocobalamin. Vitamin B12 is absorbed primarily in which portion of the gastrointestinal (GI) tract? a. Stomach b. Duodenum c. Jejunum d. Ileum e. Colon

The answer is d

316. A 35-year-old male smoker presents with burning epigastric pain that is most pronounced on an empty stomach. In addition to peptic ulcer disease and gastric acid hypersecretion, the patient has a paroxysmal rise in serum gastrin in response to intravenous (IV) secretin. Normally, basal acid output is increased by which of the following? a. Acidification of the antrum b. Acidification of the duodenum c. Administration of an H2-receptor antagonist d. Alkalinization of the antrum e. Vagotomy

The answer is d

320. A 17-year-old adolescent boy who is being treated with the macro-lide antibiotic erythromycin complains of nausea, intestinal cramping, and diarrhea. The side effects are the result of the antibiotic binding to receptors in the GI tract that recognize which GI hormone? a. Cholecystokinin b. Enterogastrone c. Gastrin d. Motilin e. Secretin

The answer is d

322. A 57-year-old woman undergoes resection of the terminal ileum as part of treatment for her chronic inflammatory bowel disease. Removal of the terminal ileum will most likely result in which of the following? a. Decreased glucose absorption b. Decreased water content of the feces c. Increased bile acid concentration in the enterohepatic circulation d. Increased excretion of fatty acids e. Increased iron absorption

The answer is d

323. A 67-year-old man with a history of alcohol abuse presents to the emergency room with severe epigastric pain, hypotension, abdominal distension, and diarrhea with steatorrhea. Serum amylase and lipase are found to be greater than normal, leading to a diagnosis of pancreatitis. The steatorrhea can be accounted for by a decrease in the intraluminal concentration of which of the following pancreatic enzymes? a. Amylase b. Chymotrypsin c. Colipase d. Lipase e. Trypsin

The answer is d

329. A 42-year-old woman presents to the emergency room with right upper quadrant pain that developed after eating dinner. She is diagnosed as having cholecystitis. Which of the following would be expected with contraction of the gallbladder following a meal? a. It is inhibited by a fat-rich meal. b. It is inhibited by the presence of amino acids in the duodenum. c. It is stimulated by atropine. d. It occurs in response to cholecystokinin. e. It occurs simultaneously with the contraction of the sphincter of Oddi.

The answer is d

342. A 56-year-old man presents with postprandial diarrhea persisting since an ileal resection. The gastric surgeon suspects bile acid malabsorption. Which of the following best describes bile acid function? a. They are essentially water insoluble. b. The majority of bile acids are absorbed by passive diffusion. c. Glycine conjugates are more soluble than taurine conjugates. d. The amount lost in the stool each day represents the daily loss of cholesterol. e. The bile acid-dependent fraction of bile is stimulated by the hormone secretin.

The answer is d

347. A 47-year-old woman with hypermenorrhea develops an iron-deficiency anemia requiring iron supplements. Which of the following best describes iron digestion and absorption? a. About 100 mg of iron is absorbed per day. b. Iron is absorbed rapidly from the small intestine. c. Iron is transported into enterocytes by a ferroportin transporter on the apical membrane. d. Iron is transported in the blood bound to transferrin. e. Iron is oxidized from the ferrous to the ferric state during absorption

The answer is d

349. A patient has vomiting and severe watery diarrhea after eating spoiled shellfish. IV fluid and electrolyte replacement was started, and a stool specimen was taken, which came back positive for Vibrio cholerae. Which of the following statements best describes water and electrolyte absorption in the GI tract? a. Electrolyte absorption in the small intestine is primarily electrogenic. b. Osmotic equilibration of chyme occurs in the ileum. c. The majority of water and electrolyte absorption occurs in the jejunum. d. The small intestine and colon have similar absorptive capacities. e. The toxin produced by V. cholerae decreases cyclic AMP in intestinal epithelial cells.

The answer is d

352. A 27-year-old female patient with a history of IBS presents with a chief complaint of flatulence. Gas within the colon is primarily derived from which of the following sources? a. Air pockets in diverticula b. CO2 liberated by the interaction of bicarbonate and H+ c. Diffusion from the blood d. Fermentation of undigested oligosaccharides e. Swallowed atmospheric air

The answer is d

354. A 29-year-old internal medicine resident has a breakfast buffet after a long night of call. The rate of gastric emptying increases with an increase in which of the following? a. Acidity of duodenum b. Fat content of duodenum c. Intraduodenal volume d. Intragastric volume e. Osmolality of duodenum

The answer is d

361. After a long workout, a third-year medical student drinks a bottle of an electrolyte-containing sports drink. Which of the following is the major mechanism for absorption of sodium from the small intestine? a. Cotransport with bicarbonate b. Electrogenic transport c. Na+-K+ exchange d. Neutral NaCl absorption e. Solvent drag

The answer is d

362. A 42-year-old healthy man takes a daily multivitamin to complement his diet. Which of the following is required for absorption of the fat-soluble vitamins contained in his supplement? a. Chymotrypsin b. Intrinsic factor c. Pancreatic amylase d. Pancreatic lipase e. Secretin

The answer is d

367. Gastric emptying studies performed on a 49-year-old woman who has vomiting shortly after eating reveal a normal rate of liquid emptying, but a prolonged time for emptying of solids. Which of the following best explains these findings? a. Colonic obstruction b. Increased amplitude of antral contractions c. Inflammation of the proximal small intestine d. Pyloric stenosis e. Sectioning of the vagus nerves to the stomach

The answer is d

374. A patient with uncontrolled hypertension is placed on a new diuretic targeted to act on the Na+ reabsorption site from the basolateral surface of the renal epithelial cells. Which of the following transport processes is the new drug affecting? a. Facilitated diffusion b. Na+/H+ exchange c. Na+-glucose cotransport d. Na+-K+ pump e. Solvent drag

The answer is d

376. A 32-year-old man complaining of fatigue and muscle weakness is seen by his physician. Blood tests reveal a serum glucose level of 325 mg/dL and serum creatinine of 0.8 mg/dL. Results of a 24-hour urine analysis are as follows: Total volume = 5L Total glucose = 375 g Total creatinine = 2.4 g The patient's GFR is approximately which of the following? a. 75 mL/min b. 100 mL/min c. 125 mL/min d. 200 mL/min e. 275 mL/min

The answer is d

377. An 83-year-old woman with a history of hypertension presents to her family physician's office with oliguria. Serum creatinine and BUN are elevated and a computed tomography (CT) reveals that the patient's left kidney is hypoplastic. Renal function studies are performed to assess the renal handling of various substances. Substance X is injected into an arterial line. All of substance X appears in the urine and none is detected in the renal vein. What do these findings indicate about the renal handling of substance X? a. It must be filtered by the kidney. b. It must be reabsorbed by the kidney. c. Its clearance is equal to the GFR. d. Its clearance is equal to the renal plasma flow (RPF). e. Its urinary concentration must be higher than its plasma concentration.

The answer is d

380. A 17-year-old girl went on a starvation diet for 3 days before prom so that she would look thin in her new dress. Her mother found her lethargic and hyperventilating, and took her to the emergency department for evaluation. Based on the following laboratory values, which of the following is her net acid excretion? Plasma pH = 7.26 Urine flow = 1.2 L/day Urine bicarbonate = 2 mEq/L Urine titratable acids = 24 mEq/L Urine ammonium = 38 mEq/L Urine pH = 5.4 a. 60 mEq/L b. 64 mEq/L c. 68 mEq/L d. 72 mEq/L e. 76 mEq/L

The answer is d

397. A 47-year-old woman presents for her annual physical examination. A year ago, the patient started a diet and exercise regimen when her blood pressure was 130/85 mm Hg. She has lost 10 lb and reduced her BMI to 25 kg/m2, but her blood pressure on this visit is found to be 145/98 mm Hg. The patient is started on a combination of a low dose of hydrochlorothiazide with the K+ -sparing diuretic, triamterene. The amount of potassium excreted by the kidney will decrease if which of the following occurs? a. Circulating aldosterone levels increase. b. Dietary intake of potassium increases. c. Distal tubular flow increases. d. Na+ reabsorption by the distal nephron decreases. e. The excretion of organic ions decreases

The answer is d

403. A patient undergoing surgery develops an increase in the secretion of ACTH, cortisol, and aldosterone. Which of the following statements best characterizes the actions of aldosterone in the kidney? a. It increases hydrogen ion reabsorption from the distal convoluted tubules. b. It increases potassium ion reabsorption from the distal convoluted tubules. c. It increases sodium ion reabsorption in the proximal tubule. d. It increases the number of active epithelial sodium channels (ENaCs) in the collecting ducts. e. It produces its effect by activating cyclic adenosine monophosphate (cAMP).

The answer is d

408. A 38-year-old woman is admitted to the hospital by her physician because of decreased urine output. Prior to admission, she was rehearsing for a dance performance and had been taking ibuprofen (Motrin) for pain. Laboratory data reveal the following: BUN, 49 mg/dL; serum sodium, 135 mmol/L; serum creatinine, 7.5 mg/dL; urine sodium, 33 mmol/L, and urine creatinine, 90 mg/dL. Her fractional sodium excretion is approximately which of the following? a. 0.5% b. 1.0% c. 1.5% d. 2.0% e. 3.0%

The answer is d

411. A 58-year-old man is hospitalized following an acute myocardial infarction. Several days later, the patient's 24-hour urine output is lower than normal. An increase in which of the following contributes to a reduced urine flow in a patient with congestive heart failure and reduced effective circulating volume? a. ANP b. Renal natriuretic peptide (urodilatin) c. Renal perfusion pressure d. Renal sympathetic nerve activity e. Sodium delivery to the macula densa

The answer is d

412. A 58-year-old man presents with hematuria, abdominal pain, and fatigue. Physical examination reveals a flank mass and an abdominal CT reveals a large solid mass on the left kidney. Laboratory studies show anemia and increased creatinine and BUN suggestive of advanced disease. A decrease in GFR would result from which of the following? a. A decrease in the concentration of plasma protein b. An increase in afferent arteriolar pressure c. An increase in RBF d. Compression of the renal capsule e. Constriction of the efferent arteriole

The answer is d

426. A 39-year-old man presents with severe writhing back pain, hematuria, and nausea. An intravenous pyelogram confirms a diagnosis of renal calculi. The presence of strongly opaque stones on the plain film is suggestive of calcium oxalate stones, which have an increased incidence with hypophosphatemia. The renal clearance of phosphate is increased by which of the following hormones? a. Aldosterone b. Angiotensin c. Norepinephrine d. Parathyroid hormone (PTH) e. Vasopressin

The answer is d

427. A 41-year-old woman presents with hemoptysis and hematuria. Laboratory findings include markedly elevated BUN, creatinine, and erythrocyte sedimentation rate. Serum is positive for antiproteinase-3 ANCA, and negative for antiglomerular basement membrane antibody, suggesting Wegener granulomatosis rather than Goodpasture disease. Urinalysis reveals proteinuria and RBC casts, in addition to the hematuria. Progressive renal failure makes it difficult for the patient to excrete a normal dietary potassium load. Which of the following will produce the greatest increase in potassium secretion? a. A decrease in circulating blood volume b. A decrease in renal blood flow c. A decrease in urine flow rate d. An increase in distal nephron sodium concentration e. An increase in sympathetic nerve activity

The answer is d

433. A 65-year-old male with a past medical history of COPD, hypertension, diabetes mellitus type II, and hypercholesterolemia presents to the emergency department with swelling of the legs and feet and shortness of breath. After complete history, physical exam, and appropriate diagnostic testing, it is determined that the patient is experiencing volume overload as a result of an acute exacerbation of congestive heart failure. The patient is started on oxygen therapy, nebulizer treatments, and intravenous furosemide, which increases sodium reabsorption in the thin ascending limb of the loop of Henle via which of the following mechanisms? a. Na+Cl− cotransport b. Na+/H+ exchange c. Na+/K+ exchange d. Na+− K+ − 2Cl − cotransport e. Na+ /nutrient cotransport

The answer is d

436. A 15-year-old boy presents for his annual athletic physical exam. A thorough examination reveals unilateral cryptorchidism. The physician schedules a follow-up visit with the boy and his parents to discuss his recommendation for surgery to correct the defect because of his concerns of possible infertility in the future. Which of the following best describes spermatogenesis? a. Leydig cell secretion of testosterone requires follicle-stimulating hormone (FSH). b. Luteinizing hormone (LH) acts directly on Sertoli cells to promote cell division. c. Mature spermatozoa are present at birth, but cannot be released until puberty is reached. d. Spermatogenesis requires a temperature lower than internal body temperature. e. Spermatogenesis requires continuous release of gonadotropin-releasing hormone (GnRH).

The answer is d

439. A 22-year-old woman presents to the obstetrician-gynecologist's office with complaints of painful menstruation accompanied by profuse menstrual flow. The doctor prescribes ibuprofen and an oral contraceptive. Biological actions of estrogens include a decrease in which of the following? a. Duct growth in the breasts b. Libido c. Ovarian follicular growth d. Serum cholesterol levels e. Uterine smooth muscle motility

The answer is d

442. A 23-year-old woman who has been training for a marathon presents with secondary amenorrhea. A pregnancy test is negative, so the woman is started on the orally active opioid blocker naltrexone, which restores ovulation and menses. Ovulation is caused by a sudden increase in the secretion of which of the following hormones? a. Estrogen b. FSH c. GnRH d. LH e. Progesterone

The answer is d

464. A 33-year-old major league baseball player takes human growth hormone to increase his performance. Which of the following best describes human growth hormone? a. It decreases lipolysis. b. It has a long half-life. c. It inhibits protein synthesis. d. It stimulates production of somatomedins (insulin-like growth factors I and II [IGF-I and II]) by the liver, cartilage, and other tissues. e. Its secretion is stimulated by somatostatin and inhibited by ghrelin.

The answer is d

474. A 50-year-old alcoholic man presents with cirrhotic liver disease and chronic pancreatitis. He has been experiencing nausea for the past several days, and not eating. As a result of an elevation in his blood glucagon levels, which of the following will occur? a. Inhibition of adenylate cyclase b. Inhibition of insulin secretion c. Inhibition of phospholipase C d. Stimulation of gluconeogenesis in the liver e. Stimulation of glycogenolysis in muscle

The answer is d

477. A patient with tuberculosis becomes confused and complains of muscle cramps and nausea. Laboratory results show a plasma sodium concentration of 125 mEq/L, serum osmolarity of 200 mOsm/kg, urine osmolarity of 1,500 mOsm/kg, urine sodium of 400 mEq/d, and a normal blood volume. These clinical findings are consistent with which of the following? a. Decreased secretion of aldosterone b. Decreased secretion of ADH c. Increased secretion of aldosterone d. Increased secretion of ADH e. Increased secretion of atrial natriuretic peptide

The answer is d

484. A 46-year-old woman on lithium therapy for her bipolar disorder presents with complaints of weakness, arthralgia, and constipation. Blood work reveals hypercholesterolemia, increased levels of TSH, and decreased free T4 levels. Which of the following is also likely to be associated with her hypothyroid state? a. Decreased body mass index b. Heat intolerance c. Increased metabolic rate d. Sleepiness e. Tachycardia

The answer is d

491. A 57-year-old postmenopausal woman takes calcium and vitamin D supplements daily to prevent osteoporosis. Which of the following best describes vitamin D? a. 1,25 (OH)2-vitamin D production increases when PTH secretion decreases. b. It is a water-soluble vitamin. c. It is converted to 1,25 (OH)2-vitamin D in the liver. d. The physiologically active form of vitamin D is 1,25 (OH)2-vitamin D. e. Vitamin D deficiency is seen in areas with high sun exposure.

The answer is d

496. An 18-year-old man with hemophilia A suffered multiple internal injuries from a motorcycle accident. He is now presenting with dizziness, abdominal pain, dark patches on his elbows and knees, and cravings for chips and french fries. He is referred to an endocrinologist who makes the diagnosis of Addison's disease, and prescribes cortisol. Cortisol administration to a patient with adrenal insufficiency will result in which of the following? a. Enhanced wound healing b. Increased ACTH secretion c. Increased corticotropin-releasing hormone (CRH) secretion d. Increased gluconeogenesis e. Increased insulin sensitivity in muscle

The answer is d

497. An 80-year-old man reports increasing dyspnea, which worsens with exertion. The cardiologist orders an echocardiogram, brain natriuretic peptide, and atrial natriuretic peptide (ANP) to evaluate possible congestive heart failure. Which of the following is most likely with ANP? a. ANP acts only on the distal nephron to increase urine flow. b. ANP constricts afferent renal arterioles. c. ANP enhances ADH secretion. d. ANP secretion increases when central venous pressure increases. e. ANP secretion is stimulated by hyponatremia

The answer is d

499. Sulfonylurea treatment in a 53-year-old type 2 diabetic patient causes a fall in the patient's plasma glucose concentration to 45 mg/dL. Which of the following is a sign and symptom of hypoglycemia? a. Bradycardia b. Dry skin c. Insomnia d. Loss of fine motor skills e. Satiety

The answer is d

73. A 26-year-old woman presents at the obstetrician's office for her second trimester evaluation. Which of the following values would normally be less in the fetus than in the mother? a. Affinity of Hb for oxygen b. Cardiac glycogen content c. Cardiac output/kg body weight d. Erythrocyte binding of 2,3-bisphosphoglycerate e. Hb concentration

The answer is d

77. A 9-year-old African American boy is brought to the emergency department by his mother who states that he was complaining of muscle aches and pain while playing basketball, which became worse whenever he was running up and down the court. She reports that he was sick with a fever last week, but she thought he was feeling better so she let him go to his summer basketball camp. Blood tests show anemia, increased reticulocyte count, and crescent-shaped cells. Hb electrophoresis confirms the presence of HbS. The primary mechanism for the change in RBC shape during a sickle cell crisis is which of the following? a. A decrease in erythrocyte volume during dehydration b. A rightward shift in the oxyhemoglobin dissociation curve of HbS compared with normal c. Low levels of erythropoietin d. Polymerization of HbS as it is deoxygenated e. The presence of antibodies against the red blood cell membrane

The answer is d

78. A 67-year-old man with chronic bronchitis is brought to the emergency department exhibiting labored breathing and cyanosis. The presence of cyanosis is due to which of the following? a. Decreased O2 unloading at the tissue capillary level b. Decreased oxyhemoglobin content in the capillary blood c. Decreased total arterial oxygen content d. Increased concentration of deoxygenated Hb e. Increased hematocrit

The answer is d

81. An 82-year-old woman with end-stage renal disease is brought to the emergency department complaining of nausea, vomiting, muscle cramps, and generalized weakness. Laboratory analysis reveals significant hyper-kalemia. Elevations of extracellular potassium ion concentration will have which of the following effects on nerve membranes? a. The activity of the Na+-K+pump will decrease b. The membrane potential will become more negative c. The membrane will become more excitable d. Potassium conductance will increase e. Sodium conductance will increase

The answer is d

89. A 41-year-old man is seen by his physician complaining of "always feeling tired" and having "vivid dreams when he is sleeping." He is referred to the hospital's sleep center for evaluation. He is diagnosed with narcolepsy based on his clinical history and the presence of rapid eye movements (REM) as soon as he falls asleep. Which of the following signs will be observed when the patient is exhibiting REM sleep? a. High-amplitude electroencephalogram (EEG) waves b. Hyperventilation c. Low frequency EEG waves d. Periods of loss of skeletal muscle tone e. Slow but steady heart rate

The answer is d

100. An 86-year-old woman develops unilateral vesicular eruption on the trunk in a T8 dermatomal pattern. Staining of the skin scrapings confirms a diagnosis of herpes zoster. The woman complains of significant pain, as well as increased sensitivity to touch. Which of the following receptors is responsible for measuring the intensity of a steady pressure on the skin surface? a. Krause ending b. Meissner corpuscle c. Merkel disk d. Pacinian corpuscle e. Ruffini ending

The answer is e

102. A 48-year-old woman with multiple sclerosis (MS) and increasing spasticity is treated with an intrathecal infusion of baclofen, a GABAB agonist that mediates presynaptic inhibition. Presynaptic inhibition in the central nervous system (CNS) affects the firing rate of α-motoneurons by which of the following mechanisms? a. Decreasing the frequency of action potentials by the presynaptic nerve ending b. Decreasing the potassium permeability of the α-motoneuron c. Hyperpolarizing the membrane potential of the α-motoneuron d. Increasing the amount of the neurotransmitter released by the presynaptic nerve ending e. Increasing the chloride permeability of the presynaptic nerve ending

The answer is e

107. A 34-year-old woman, who has been immobilized with a sprained ankle for the past 4 days, develops a throbbing pain that has spread to her entire left leg. History reveals that she has been taking oral contraceptives for 15 years. Ischemic pain is associated with which of the following? a. A beta (Aβ) sensory fibers b. Afferent pathway in the neospinothalamic tract c. Overstimulation of somatic touch receptors d. Rapidly adapting receptors e. Sensory fibers terminating within the substantia gelatinosa of the dorsal horn of the spinal cord

The answer is e

112. A 27-year-old man with severe epilepsy, characterized by major convulsions and lapses of consciousness every few minutes, underwent experimental neurosurgery to help relieve his seizures. The operation had a significant, beneficial effect on the epilepsy, but led to a devastating memory deficit. He had normal procedural memory, maintained long-term memory for events that occurred prior to surgery, and his short-term memory was intact, but he could not commit new events to long-term memory (loss of declarative memory). Which of the following areas of the brain was bilaterally resected in this patient? a. Cerebral cortex b. Cingulate gyrus c. Hypothalamus d. Parietal lobe e. Temporal lobe

The answer is e

115. A 58-year-old woman goes to her physician because she is having difficulty threading needles. An eye examination leads to the diagnosis of presbyopia (old eyes). Her condition is most likely caused by which of the following? a. Ciliary muscle paralysis b. Clouding of the vitreous c. Degeneration of the macula d. Retinal detachment e. Stiffening of the lens

The answer is e

119. A 52-year-old man has a motor vehicle accident and closed head injury, after which he notes decreased sense of smell. Anosmia after head injury is most likely associated with which of the following? a. Direct injury to the olfactory bulbs b. Fractures of the cribriform plate c. Low Glasgow coma scale d. Recovery of olfaction over time to baseline e. Shearing of the olfactory fila as they pass through the cribriform plate

The answer is e

122. A 26-year-old African American female medical student goes to the emergency department when she sees flashes of light, moving spots, and has reduced visual acuity. An ophthalmology consult reveals that she is myopic, does not have eye pain, and has a scotoma in the peripheral vision field of her right eye. There is no cherry red spot on the fovea. Which of the following is the most likely cause for her acute vision loss? a. Central retinal artery embolism b. Glaucoma c. Macular degeneration d. Optic neuritis e. Retinal detachment

The answer is e

125. A 32-year-old woman from the IT department presents to the employee health clinic late in the afternoon complaining of fatigue, muscular weakness, and double vision. She indicates that the symptoms have been getting worse over the past 2 months and that she gets worse the longer she works at the computer screen. Cranial nerve examination discloses impaired movement of the right eye and bilateral ptosis, which worsen with repetitive eye movements. An MRI of the chest shows enlargement of the thymus gland. The neuropathy of this clinical presentation is most likely caused by antibodies against which of the following? a. Acetylcholine b. Acetylcholinesterase c. Postsynaptic muscarinic acetylcholine receptors d. Presynaptic nicotinic acetylcholine receptors in autonomic ganglia e. Postsynaptic nicotinic acetylcholine receptors on the motor end plate

The answer is e

129. A fireman suffers extensive burns, resulting in a fluid and electrolyte imbalance. Which of the following electrolyte imbalances will produce a decrease in the magnitude of a nerve membrane action potential? a. Hyperkalemia b. Hypernatremia c. Hypocalcemia d. Hypokalemia e. Hyponatremia

The answer is e

136. A 19-year-old sexually active woman presents with lower abdominal pain for 1 week. Physical examination reveals a temperature of 101°F (38.33°C), tendernesson pelvic examination, and a mucopurulent vaginal discharge. Synaptic transmission between pain fibers from the pelvis and spinal cord neurons is mediated by which of the following? a. Acetylcholine b. Endorphins c. Serotonin d. Somatostatin e. Substance P

The answer is e

141. A 62-year-old man presents with a hypokinetic movement disorder characterized by paucity and slowness of voluntary movement (bradykinesia). Which of the following characterizes the order of recruitment during normal voluntary movement? a. Anaerobic fibers are recruited before aerobic fibers b. Fast muscle fibers are recruited before slow muscle fibers c. Large muscle fibers are recruited before small muscle fibers d. Poorly perfused muscle fibers are recruited before richly perfused muscle fibers e. Weak muscle fibers are recruited before strong muscle fibers

The answer is e

168. An insulation worker presents with a chief complaint of dyspnea on exertion. Pulmonary function test is consistent with a restrictive impairment. His arterial PO2 is normal at rest but hypoxemic during exercise stress testing. Which of the following is the most likely explanation for the decline in the patient's PaO2 during exercise compared with rest? a. A decreased partial pressure gradient for O2 diffusion during exercise b. A decreased surface area for diffusion during exercise c. An increase in hemoglobin's affinity for O2 during exercise resulting in more oxygen being transported as oxyhemoglobin and less in the dissolved state d. An increased uptake of oxygen from the blood by exercising skeletal muscles e. An underlying diffusion impairment coupled with a decrease in pulmonary capillary transit time during exercise

The answer is e

188. A 35-year-old woman with gestational diabetes develops hypertension and preeclampsia, requiring the preterm delivery of her fetus of 30 weeks of gestation. The woman is given two doses of betamethasone, 12 mg, intramuscularly, 24 hours apart. Which of the following is the purpose of prenatal steroid therapy? a. Increase blood flow from the right atrium into the left atrium across the foramen ovale b. Increase blood flow to the fetal lungs c. Increase fetal PO2 d. Shift the fetal oxyhemoglobin dissociation curve to the right e. Increase the lecithin/sphingomyelin ratio in the amniotic fluid

The answer is e

194. An 18-year-old male college freshman living in a dormitory contracts meningitis, which causes a centrally mediated increase in his respiratory rate. The pacemaker neurons responsible for respiratory rhythmogenesis are located in which of the following regions of the brain? a. Apneustic center in the pons b. Central chemoreceptors in the medulla c. Inspiratory neurons in the dorsal respiratory group d. Pontine respiratory groups e. Pre-Bötzinger complex in the ventral respiratory group

The answer is e

205. An 18-year-old man is life-flighted to a Level 1 trauma center after being thrown from his motorcycle. It is determined that he has a brain tran-section above the pons. How will this lesion affect the control of breathing in this patient? a. All breathing movements will cease. b. The central chemoreceptors will no longer be able to exert any control over ventilation. c. The peripheral chemoreceptors will no longer be able to exert any control over ventilation. d. The Hering-Breuer reflex will be abolished. e. The limbic system will no longer be able to exert any control over ventilation.

The answer is e

217. A 49-year-old farmer develops headache and becomes dizzy after working on a tractor in his barn. His wife suspects carbon monoxide poisoning and brings him to the emergency department where he complains of dizziness, lightheadedness, headache, and nausea. The patient's skin is red, he does not appear to be in respiratory distress, and denies dyspnea. Blood levels of carboxyhemoglobin are elevated. Which of the following best explains the absence of respiratory signs and symptoms associated with carbon monoxide poisoning? a. Blood flow to the carotid body is decreased b. Arterial oxygen content is normal c. Cerebrospinal fluid (CSF) pH is normal d. Central chemoreceptors are depressed e. Arterial oxygen tension is normal

The answer is e

218. A 68-year-old patient with shortness of breath is referred for pulmonary function testing, including lung volumes, flow-volume curves, and lung compliance. Which of the following statements best characterizes lung compliance? a. It decreases with advancing age. b. It increases when there is a deficiency of surfactant. c. It increases in patients with pulmonary edema. d. It is equivalent to ∆P/∆V. e. It is inversely related to the elastic recoil properties of the lung.

The answer is e

220. A patient complains of paroxysmal episodes of not being able to catch her breath. When no abnormalities are detected with conventional pulmonary function screening, the pulmonologist orders a methacholine challenge test. Which of the following will increase as a result of stimulating cholinergic receptors on the bronchial smooth muscle? a. Airway diameter b. Anatomic dead space c. Compliance of the lungs d. Elastic work of breathing e. Resistive work of breathing

The answer is e

227. A 78-year-old woman presents to her family physician's office with a chief complaint of fatigue and shortness of breath. The doctor indicates that he wants her to go to the hospital to get some pulmonary function tests, but there is one who is able to do in the office. A spirometer can be used to directly measure which of the following? a. FRC b. Peak flow rate c. Residual volume d. Total lung capacity e. Vital capacity

The answer is e

231. A 56-year-old woman presents for her annual physical examination. Her physician auscultates a late systolic crescendo murmur with a midsystolic click. The murmur is best heard over the apex, is loudest at S2, is shortened with squatting, and is longer and more intense when venous return is decreased by standing or a Valsalva maneuver. Which of the following values is the best index of the preload on her heart? a. Blood volume b. Central venous pressure c. Pulmonary capillary wedge pressure d. Ventricular end-diastolic pressure e. Ventricular end-diastolic volume

The answer is e

243. A 75-year-old woman presents with fatigue and orthopnea. Rales are heard widely over both lung fields. After several days of furosemide to treat the patient's fluid retention, lisinopril therapy is initiated. Which of the following is primarily responsible for the improvement in her condition with lisinopril? a. An increase in cardiac contractility b. An increase in ventricular end-diastolic pressure c. An increase in ventricular wall thickness d. A reduction in heart rate e. Stabilization of cardiac remodeling

The answer is e

244. A 37-year-old woman undergoes a CT scan of the abdomen, which reveals a large peritoneal mass. A subsequent magnetic resonance angiography study showed that the abdominal aorta was constricted to one-half of its resting diameter. As a result, resistance to blood flow through the vessel would be which of the following? a. Decreased in half b. Decreased 16-fold c. Doubled d. Increased by 50% e. Increased 16-fold

The answer is e

254. Physical examination of a 41-year-old IV drug abuser reveals an early systolic murmur. The physician also notes a 7-cm distance between the height of the blood in his right internal jugular vein and sternal angle (normal = 3 cm). Which of the following conditions is most likely responsible for the physical findings? a. Aortic regurgitation b. Atherosclerosis c. Mitral stenosis d. Tachycardia e. Tricuspid regurgitation

The answer is e

259. A 19-year-old man severs an artery in a motorcycle accident. A bystander applies a tourniquet to stop the bleeding. When the paramedics arrive, the blood pressure of the injured man was only slightly hypotensive and his pupils were reactive. The greatest percentage of the redistributed blood volume came from which of the following? a. Aorta b. Arteries and arterioles c. Capillaries d. The heart e. Venules and veins

The answer is e

260. An 84-year-old woman presents with paroxysmal dizziness, syncope, confusion, and fatigue. Her heart rate did not change when the patient was instructed to perform a Valsalva maneuver. A 24-hour Holter monitor revealed periodic episodes of sinus bradycardia. Phase 4 of the pacemaker potential of SA nodal cells is caused by which of the following? a. A decrease in the activity of the Na-K pump b. A decrease in the flow of potassium out of the cell c. An increase in the activity of the Na/Ca exchanger d. An increase in the flow of calcium into the cell e. An increase in the flow of sodium into the cell

The answer is e

263. An 83-year-old woman with long-standing hypertension presents after a near-syncopal episode upon standing. Her blood pressure is taken sitting and then standing. Systolic pressure decreased slightly and pulse pressure increased in the standing position. Which of the following can lead to an increased pulse pressure? a. A decrease in stroke volume b. An increase in arterial compliance c. An increase in heart rate d. An increase in total peripheral resistance e. Stiffening of the arteries

The answer is e

265. A 68-year-old sedentary, obese man with four-vessel coronary occlusive disease has a massive myocardial infarction while shoveling snow. In the blizzard conditions, it takes the ambulance over an hour to reach the man's home. When the paramedics arrive, the patient's radial pulse is rapid and thready, he has pink froth coming from his mouth, and he is nonresponsive. Increasing which of the following would lead to an increased stroke volume in this patient? a. Heart rate b. Pulmonary capillary hydrostatic pressure c. Systemic vascular resistance d. Venous compliance e. Ventricular contractility

The answer is e

268. A patient comes to his physician complaining that he is no longer able to exercise as long as he used to. The physician auscultates crepitant rales and a third heart sound; blood pressure is normal. He sends the patient to cardiologist because of suspected heart failure. Which of the following is most consistent with a diagnosis of CHF? a. Decreased heart rate b. Decreased left ventricular energy consumption c. Decreased pulmonary arterial wedge pressure d. Increased left ventricular ejection fraction e. Increased left ventricular wall tension

The answer is e

272. A 67-year-old man with a history of rheumatic heart disease presents with difficulty breathing when he exercises, which has worsened over the last year. Auscultation reveals a holosystolic murmur at the left 5th intercostal space along the midclavicular line. The murmur is loudest at the apex, radiates to the axilla, and is enhanced during expiration and when the patient is instructed to make a fist. Which of the following findings is most likely to be present? a. Decreased arterial pressure b. Decreased left ventricular preload c. Increased cardiac output d. Increased pulse pressure e. Increased ν wave

The answer is e

281. A 2-year-old boy is mauled by a black bear while hiking with his family in the Appalachian Mountains. A claw-puncture wound to the skull compressed the underlying brain tissue. Which of the following occurs in response to an increased intracranial pressure? a. Blood pressure and heart rate decrease. b. Blood pressure and heart rate increase. c. Blood pressure and heart rate remain constant. d. Blood pressure decreases and heart rate increases. e. Blood pressure increases and heart rate decreases.

The answer is e

304. A 22-year-old woman is recovering from an upper respiratory infection with Coxsackie B virus when her condition worsens and she becomes increasingly dyspneic. An echocardiogram reveals global hypokinesis and an ejection fraction of 25%. Which of the following is the underlying process in this patient's most likely diagnosis? a. Global ventricular hypertrophy b. Hypertrophy of the interventricular septum c. Narrowing of the descending aorta d. Vegetations on the cardiac valves e. Ventricular dilation

The answer is e

308. A 58-year-old woman with idiopathic pulmonary hypertension presents with right ventricular hypertrophy and cor pulmonale. Her ECG shows positive QRS complexes in leads V1, III, and aVF, and equiphasic QRS complexes in lead aVR. Which of the following is her mean QRS vector? a. −150 degrees b. −120 degrees c. +60 degrees d. +90 degrees e. +120 degrees

The answer is e

318. A 37-year-old man with AIDS presents with a fever, anorexia, weight loss, and GI bleeding. Physical examination reveals a palpable abdominal mass. Endoscopy and biopsy reveal a proximal small-bowel malignancy requiring surgical resection. Removal of proximal segments of the small intestine would most likely result in a decrease in which of the following? a. Basal acid output b. Maximal acid output c. Gastric emptying of liquids d. Gastric emptying of solids e. Pancreatic enzyme secretion

The answer is e

330. A 42-year-old airline pilot presents to his family physician with a chief complaint of mid-epigastric pain that is relieved by antacids or eating. Endoscopic evaluation reveals the presence of a duodenal ulcer. Based on the diagnosis, which of the following also would be expected? a. Decreased basal acid output b. Decreased gastric emptying of liquids c. Decreased gastric emptying of solids d. Increased bicarbonate secretion in the duodenal bulb e. Increased maximal acid output

The answer is e

334. A 47-year-old man takes esomeprazole for his "acid indigestion." Which of the following best describes the use of substituted benzimidazole derivatives? a. They are not effective as part of a treatment regimen for Helicobacter pylori. b. They are not used in the treatment of Zollinger-Ellison syndrome. c. They are water-soluble compounds. d. They decrease gastrin levels. e. They inhibit H-K-ATPase in parietal cells.

The answer is e

336. A 62-year-old woman is prescribed the prostaglandin E agonist, misoprostol, along with a NSAID for her severe bilateral osteoarthritis of the knees. What is the purpose of the misoprostol? a. Antagonizes H2 receptors b. Enhances LES relaxation, thereby preventing NSAID-induced dysphagia c. Increases colonic water reabsorption, thereby preventing diarrhea d. Inhibits bicarbonate secretion into the gastric mucous gel e. Prevents NSAID-induced gastric ulcers

The answer is e

338. A 32-year-old woman presents to the emergency department with abdominal pain and diarrhea accompanied by steatorrhea. Gastric analysis reveals a basal acid level of 12 mmol/hour (normal: <5 mmol/hour). The steatorrhea is most likely due to which of the following? a. Decreased gastric acid secretion b. Decreased pyloric sphincter tone c. Decreased secretion of intrinsic factor d. Delayed gastric emptying e. Inactivation of pancreatic lipase due to low duodenal pH

The answer is e

339. A 42-year-old obese woman presents to the emergency department with right upper quadrant pain, nausea, and vomiting. The pain is not related to food intake and lasts for several hours before resolving slowly. Ultrasound images are suggestive of gallstones with cystic duct obstruction. Which of the following is the primary physiological stimulus of gall-bladder contraction in the digestive period? a. Acid-induced release of motilin from the small intestine b. Acid-induced release of secretin from the small intestine c. Amino acid-induced release of motilin from the small intestine d. Distension-induced release of glucagon from the small intestine e. Fat-induced release of cholecystokinin from the small intestine

The answer is e

341. A 31-year-old man presents to the emergency department with the symptoms of heartburn and difficulty swallowing. Esophageal manometry reveals an inflamed esophageal mucosa and a hypotensive LES. A diagnosis of GERD is made and the patient is subsequently treated with a proton pump inhibitor. Normally, which of the following is most likely regarding reflux of gastric acid into the esophagus? a. It inhibits esophageal bicarbonate secretion. b. It inhibits gastric acid secretion. c. It inhibits gastric motility. d. It initiates primary esophageal peristalsis. e. It initiates secondary esophageal peristalsis.

The answer is e

348. A patient with alcoholic cirrhosis presents to the emergency room with hematemesis. After stabilizing him with IV fluids, the gastroenterolo-gist administers an analog of which of the following agents to inhibit gastric acid secretion and visceral blood flow? a. Acetylcholine b. Gastrin c. Histamine d. Pepsin e. Somatostatin

The answer is e

359. A 47-year-old woman presents to her primary care physician with jaundice. She is found to have elevated levels of direct (conjugated) plasma bilirubin. Which of the following is the most likely diagnosis? a. Crigler-Najjar syndrome type I b. Crigler-Najjar syndrome type II c. Gilbert syndrome d. Hemolytic anemia e. Obstruction of the common bile duct

The answer is e

363. Following gastric bypass surgery, a patient presents with crampy abdominal discomfort 15 to 30 minutes after meals, accompanied by nausea, diarrhea, belching, tachycardia, palpitations, diaphoresis, and light-headedness. These symptoms most likely arise from which of the following? a. Decreased emptying of hyperosmotic gastric contents b. Decreased insulin release c. Hyperglycemia d. Increased secretion of cholecystokinin e. Release of VIP and motilin

The answer is e

364. A 61-year-old woman presents with diarrhea, abdominal pain, and flushing. Urinary excretion of the serotonin metabolite, 5-hydroxyindoleacetic acid, is elevated. Abdominal CT reveals a tumor in the terminal ileum. Surgical resection of the terminal ileum will most likely result in which of the following? a. A decrease in absorption of amino acids b. A decrease in the fat content of the feces c. An increase in the absorption of iron d. An increase in the concentration of bile acid in the enterohepatic circulation e. An increase in the water content of the feces

The answer is e

372. A previously well 12-year-old boy is brought to the emergency department with vomiting and severe abdominal cramps after a prolonged period of exercise. Elevated levels of serum creatinine and blood urea nitrogen (BUN) suggest acute renal failure. Following treatment and recovery, his serum uric acid concentration (0.6mg/dL) remains consistently below normal. To determine if his low serum uric acid level is related to renal dysfunction, uric acid clearance studies are conducted and the following data are obtained: Urine flow rate = 1 mL/min Urine [uric acid] = 36 mg/dL Which of the following is the patient's uric acid clearance? a. 6 mL/min b. 12 mL/min c. 24 mL/min d. 48 mL/min e. 60 mL/min

The answer is e

373. A 69-year-old man presents with symptoms of thirst and dizziness, and physical evidence of orthostatic hypotension and tachycardia, decreased skin turgor, dry mucous membranes, reduced axillary sweating, and reduced jugular venous pressure. He was recently placed on an angiotensin-converting enzyme (ACE) inhibitor for his hypertension. Urinalysis reveals a reduction in the fractional excretion of sodium and the presence of acellular hyaline casts. The internist suspects acute renal failure of prerenal origin associated with increased renin secretion by the kidney. A stimulus for increasing renal renin secretion is an increase in which of the following? a. Angiotensin II b. Atrial natriuretic peptide (ANP) c. GFR d. Mean blood pressure e. Sympathetic nerve activity

The answer is e

387. A 52-year-old man presents to his internist for a 6-month checkup following diuretic therapy and recommended diet changes for his essential hypertension. His blood pressure is 145/95 mm Hg and serum aldosterone levels are increased. Aldosterone secretion is increased when there is a decrease in the plasma concentration of which of the following? a. Adrenocorticotropic hormone (ACTH) b. Angiotensin II c. Potassium d. Renin e. Sodium

The answer is e

388. A 76-year-old man presents at the emergency department with headache, vomiting, shortness of breath, insomnia, and confusion. He is found to be oliguric with an increased BUN and creatinine. Urine-specific gravity is low and there is proteinuria. Which of the following statements concerning the normal renal handling of proteins is correct? a. Proteins are more likely to be filtered if they are negatively charged than if they are uncharged. b. Proteins can be filtered and secreted but not reabsorbed by the kidney.c. Most of the protein excreted each day is derived from tubular secretion. d. Protein excretion is directly related to plasma protein concentration. e. Protein excretion is increased by sympathetic stimulation of the kidney.

The answer is e

392. A patient with multiple myeloma develops type II RTA with a defect in proximal tubular bicarbonate reabsorption. Which of the following structural features distinguishes the epithelial cells of the proximal tubule from those of the distal convoluted tubule? a. The distal tubule has a thicker basement membrane. b. The distal tubule has fewer tight intercellular junctions. c. The distal tubule has more microvilli. d. The proximal tubule forms the juxtaglomerular apparatus. e. The proximal tubule has a more extensive brush border.

The answer is e

394. A 56-year-old man with hypertension presents with complaints of flushing and orthostatic hypertension. Blood analysis reveals an increased plasma renin activity and hyperlipidemia. Urinalysis reveals a decreased GFR and an increase in urinary albumin excretion. Gadolinium-enhanced three-dimensional magnetic resonance angiography is suggestive of renal artery stenosis. Measurement of RBF and a renal arteriogram are ordered to evaluate the patient for atherosclerotic renal vascular disease (ARVD = renal artery stenosis and ischemic nephropathy). The effective RPF, determined from the clearance of PAH, is less than the true RPF because of which of the following? a. The calculated clearance of PAH depends on the urinary flow rate. b. The cortical and medullary collecting ducts are able to reabsorb some PAH. c. The fraction of PAH filtered is less than the filtration fraction. d. The measured value of the plasma PAH concentration is less than the actual PAH concentration. e. The plasma entering the renal vein contains a small amount of PAH.

The answer is e

404. A 44-year-old African American woman with a medical history of hypertension and diabetes mellitus presents to the clinic for her routine examination. At her last visit, her blood pressure was 150/95 despite trying to control her blood pressure with diet and exercise, so the patient was placed on lisinopril. She also takes metformin for her diabetes. Today, the patient's blood pressure has decreased to 130/80. Routine laboratory work indicates that she has a blood glucose of 120 mg/dL, serum creatinine of 1.0 mg/dL, and urinary microalbumin of <30 mg. What effect does angiotensin II have on the glomerular filtration rate (GFR)? a. Decreases GFR because of constriction of the afferent arteriole b. Decreases GFR because of dilation of the efferent arteriole c. No change in GFR because of equal constriction of the afferent and efferent arteriole d. Increases GFR because of dilation of the afferent arteriole e. Increases GFR because of constriction of the efferent arteriole

The answer is e

406. A trauma patient with multiple rib fractures requires intubation and mechanical ventilation. Mechanical ventilation causes an increase in the patient's vasopressin secretion and plasma levels. Which of the following is the effect of vasopressin on the kidney? a. Increased diameter of the renal artery b. Increased glomerular filtration rate c. Increased excretion of Na+ d. Increased excretion of water e. Increased permeability of the collecting ducts to water

The answer is e

407. A 16-year-old pregnant girl is admitted to the hospital in labor. Her blood pressure is 130/85 mm Hg, and her plasma creatinine is 2.7 mg/dL (normal 0.6 to 1.2mg/dL). Renal ultrasonography demonstrates severe bilateral hydronephrosis. Which of the following is the most likely cause of this patient's high creatinine levels? a. Coarctation of the renal artery b. Hypovolemia c. Hyperproteinemia d. Increased sympathetic nerve activity e. Ureteral obstruction

The answer is e

414. A 27-year-old graduate student from China presents at the Student Medical Center for mandatory tuberculosis screening. Quantiferon testing is positive and physical examination reveals cough, cachexia, and mild respiratory distress. Chest x-ray reveals a cavitary lesion in the right upper lobe. Blood analysis reveals a serum sodium of 118 mg/dL and an increased ADH concentration. As a result, the permeability of the collecting duct will be increased to which of the following? a. Ammonium ion b. Hydrogen ion c. Potassium ion d. Sodium ion e. Urea

The answer is e

419. A 55-year-old hypertensive patient is placed on a potassium-sparing diuretic. Which of the following mechanisms of action are characteristic of potassium-sparing diuretics? a. Inhibition of H+ secretion in the proximal tubule b. Inhibition of Na-Cl cotransport in the early portion of the distal tubule c. Inhibition of Na-K-2Cl cotransport in the medullary thick ascending limb of loop of Henle d. Inhibition of Na+ reabsorption via Na channels in the collecting tubules e. Inhibition of vasopressin secretion

The answer is e

428. A 36-year-old man suffers third-degree burns over 70% of his body while responding to a three-alarm fire. His effective circulating volume and renal perfusion pressure drop precipitously and the concentration of NaCl in the intraluminal fluid in the kidney decreases. These conditions cause the juxtaglomerular apparatus to release which of the following hormones? a. Adenosine b. Aldosterone c. Angiotensinogen d. ADH e. Renin

The answer is e

455. A 32-year-old woman presents at her physician's office complaining of nausea and vomiting. The history reveals that her symptoms have been present for over a month and that they seem to be worse in the morning. A urine sample is taken and shows that the woman is pregnant. Physiological changes that occur during pregnancy include which of the following? a. Decreased production of cortisol and corticosterone b. Hypercapnia c. Increased conversion of glucose to glycogen d. Increased hematocrit e. Reduced circulating gonadotropin levels

The answer is e

456. A couple presents at the fertility center with concerns that they have not been able to conceive a child. The reproductive endocrinologist evaluates the wife to be certain that she is ovulating. Which of the following is an indication that ovulation has taken place? a. A drop in body temperature b. An increase in serum estrogen levels c. An increase in serum FSH levels d. An increase in serum LH levels e. An increase in serum progesterone levels

The answer is e

460. A 49-year-old male patient with AIDS and decliningCD4 counts has an increased frequency of systemic infections and develops sick euthyroid syndrome. Which of the following would be expected with normal thyroid function? a. T4 is formed from T3 by the process of monodeiodination. b. T4 is the physiologically active hormone. c. Thyroid-stimulating hormone (TSH) is secreted from the posterior pituitary. d. TSH initiates thyroid hormone secretion via activation of nuclear receptors in thyroid gland cells. e. TSH secretion is regulated primarily by the pituitary level of T3.

The answer is e

470. A 15-year-old girl presents with loss of the outer one-third of her eyebrows. Physical examination demonstrates slight enlargement of the thyroid gland and delayed relaxation phase of deep tendon reflexes. Blood work shows an elevation in creatine phosphokinase and TSH. Thyroid hormone therapy is ordered. Physiologically active thyroxine exists in which of the following forms? a. As a glucuronide b. Bound to albumin c. Bound to globulin d. Bound to prealbumin e. Unbound

The answer is e

476. A 47-year-old woman with an anterior pituitary tumor presents with poor wound healing and hypertension. The endogenous secretion of ACTH is correctly described in which of the following statements? a. It is decreased during periods of stress. b. It is inhibited by aldosterone. c. It is stimulated by glucocorticoids. d. It is stimulated by epinephrine. e. It shows a circadian rhythm in humans.

The answer is e

482. A 37-year-old woman presents with exophthalmus and an enlarged thyroid gland. The levels of free thyroxine in her blood are elevated. Other clinical findings of Graves' disease include which of the following? a. Anorexia b. Bradycardia c. Decreased sweating d. Increased basal metabolic rate e. Increased weight gain

The answer is e

487. A patient presents with Whipple's triad, including plasma glucose <60 mg/dL, symptomatic hypoglycemia, and improvement of symptoms with administration of glucose. CT of the abdomen is suggestive of islet cell carcinoma. Which of the following best describes the islets of Langerhans? a. They are found primarily in the head of the pancreas. b. They contain six distinct endocrine cell types. c. They constitute approximately 30% of the pancreatic weight. d. They have a meager blood supply. e. They secrete insulin and glucagon

The answer is e

489. A 24-year-old woman presents with a slightly elevated blood pressure. She has high plasma levels of total T4, cortisol, and renin activity, but no symptoms or signs of thyrotoxicosis or Cushing's syndrome. Which of the following is the most likely explanation? a. She has an adrenocortical tumor. b. She has been subjected to chronic stress. c. She has been treated with ACTH and TSH. d. She has been treated with T3 and cortisol. e. She is in the third trimester of pregnancy.

The answer is e

492. A 22-year-old woman with insulin-dependent diabetes mellitus presents to the emergency department with nausea, vomiting, and a blood glucose of 600. She is found to have diabetic ketoacidosis (DKA). Which of the following is true regarding patients with DKA? a. Intracellular potassium levels are increased. b. Intravenous fluids correct the acidosis. c. Respiratory rate decreases. d. Serum potassium levels are decreased. e. Serum potassium levels are increased.

The answer is e

495. A 36-year-old male computer programmer works for a company that has just been acquired in a corporate takeover. He experiences symptoms of tachycardia, palpitations, and an irregular heartbeat, particularly at night. His plasma catecholamine levels are found to be increased, which may result from which of the following? a. Changing from the standing to the supine position b. An increase in blood glucose c. An increase in blood pressure d. An increase in blood volume e. An increase in plasma cortisol

The answer is e

498. A patient with multiple endocrine neoplasia type I (MEN I) and acromegaly is found to have a deletion of the 11q13 region of chromosome 11, a suppressor gene for growth hormone. Growth hormone excess results in which of the following? a. Decreased gluconeogenesis b. Decreased lipolysis c. Decreased protein synthesis d. Hypoglycemia e. Insulin resistance

The answer is e

66. A 67-year-old woman with a history of venous thromboembolism is placed on warfarin (Coumadin; Bristol-Myers Squibb Company, New York, NY USA) prophylactically. The blood concentration of Coumadin becomes too high and bleeding occurs. The bleeding can best be treated by the administration of which of the following? a. Fibrinogen b. Platelets c. Protein C d. Thrombin e. Vitamin K

The answer is e

69. A 65-year-old slightly cyanotic man presents to his physician complaining of pruritus and nose bleeds. A blood test reveals a hematocrit of 62%, leading to the diagnosis of polycythemia vera. Treatment includes aspirin to prevent thrombosis and periodic phlebotomy to reduce the hematocrit. The reduction in hematocrit is beneficial because it does which of the following? a. Decreases cardiac output b. Increases arterial oxygen content c. Increases arterial oxygen saturation d. Reduces blood velocity e. Reduces blood viscosity

The answer is e

72. A 42-year-old woman presents to her doctor's office with heavy menstrual bleeding for up to 2 weeks' duration for each of the past five cycles. She also reports that she has a tendency to bruise easily, and has had several episodes of epistaxis over the past couple of months. Blood analysis shows: Hb, 8 g/dL; hematocrit, 24%; MCV, 70; platelet count, 230,000/μL. Which of the following is a likely cause of her bleeding disorder? a. Aplastic anemia b. Hemophilia c. Nonsteroidal anti-inflammatory drugs d. Vitamin B12 deficiency e. von Willebrand disease

The answer is e

74. A 61-year-old man presents to his family physician with the chief complaint of frequent diarrhea accompanied by weight loss. He reports a tendency to bruise easily and laboratory data reveal a PT of 19 seconds (normal, 11-14 seconds). The bruising and prolonged PT can be explained by a decrease in which of the following vitamins? a. A b. C c. D d. E e. K

The answer is e

84. A 72-year-old man visits his physician because he finds it difficult to hold his hand steady when painting. Examination reveals a resting tremor and rigidity. The symptoms are relieved by a single dose of levodopa. This patient's neurological signs are most likely related to a lesion within which of the following? a. Caudate nucleus and putamen b. Cerebellum c. Hippocampus d. Premotor area e. Substantia nigra

The answer is e

86. A 72-year-old man develops selective loss of the large pyramidal cells in the precentral gyrus and degeneration of the corticospinal and corticobulbar projections. Other neuronal systems are spared. He is told that the progression of the disease is variable, and that the worst prognosis is about a 3-year survival. The precentral gyrus and corticospinal and corticobulbar tracts are essential for which of the following? a. Auditory identification b. Kinesthesia c. Olfaction d. Vision e. Voluntary movement

The answer is e

90. A 43-year-old woman has a chief complaint of muscle weakness. The distribution of muscle weakness and the presence of hyperactive tendon reflexes is consistent with pyramidal tract disease. Tapping the patella tendon elicits a reflex contraction of the quadriceps muscle. Which of the following occurs during the contraction of the quadriceps muscle? a. The α-motoneurons innervating the extrafusal muscle fibers decrease their rate of firing b. The α-motoneurons to the antagonistic muscles increase their rate of firing c. The γ-motoneurons innervating the intrafusal muscle fibers increase their rate of firing d. The Ia afferents from the muscle spindle increase their rate of firing e. The Ib afferents from the Golgi tendon organ increase their rate of firing

The answer is e

96. On physical examination, stroking the plantar surface of her foot produces a reflex extension of the large toe rather than the expected flexion, which indicates damage to which of the following? a. Basal ganglia b. Brain stem c. Cerebellum d. Lower motoneurons e. Upper motoneurons

The answer is e

99. A 69-year-old man with chronic hypertension presents to his physician's office. His blood pressure is 165/105 mm Hg despite treatment with a diuretic, βblocker, and an angiotensin receptor antagonist. It is decided that a fourth drug is needed for the patient's resistant hypertension, and he is prescribed the vasodilator diltiazem, a calcium channel antagonist. The effect of decreasing the resistance of the afferent arteriole in the glomerulus of the kidney is to decrease which of the following aspects of renal function? a. Filtration fraction b. Glomerular filtration rate c. Oncotic pressure of the peritubular capillary blood d. Renal plasma flow e. Renin release from juxtaglomerular cells

The answer is e

23. The friends of a 26-year-old man plan a bachelor's party for him in Las Vegas. After a round of golf, the group heads to the pool. They order several rounds of drinks over the next 4 hours and also order lunch poolside. Most of the group orders hamburgers and French fries, but the groom-to-be is watching his weight and opts for a club sandwich and a side of coleslaw. Later that night, they go for dinner and to the casinos, where they imbibe some more. Early the next morning, the groom-to-be becomes ill. He thinks it is just a hangover, but presents to the emergency department 36 hours later with persistent vomiting and orthostatic hypotension. Which of the following metabolic abnormalities are most likely present in this patient? a. Hypokalemia, hypochloremia, and metabolic alkalosis b. Hypokalemia, hypochloremia, and metabolic acidosis c. Hyperkalemia, hyperchloremia, and metabolic alkalosis d. Hyperkalemia, hyperchloremia, and metabolic acidosis e. Normal serum electrolytes and acid-base balance

the answer is a

3. An 83-year-old woman with sepsis develops multiorgan failure. Based on her blood urea nitrogen of >100 mg/dL, she is placed on continuous venovenous hemodialysis. Which of the following factors will increase the diffusive clearance of solutes across the semipermeable dialysis membrane? a. Area of the membrane increases b. Concentration gradient for the solutes decreases c. Lipid solubility of the solutes decreases d. Size of the solute molecules increases e. Thickness of the membrane increases

the answer is a

5. A 10-year-old boy sprains his ankle while running. History reveals that he has difficulty running, jumping, and keeping up with other children in races. His mother reports that she is also clumsy. Physical examination demonstrates foot drop, weakness, sensory loss, and reduced reflexes. The boy is found to have a decrease in nerve conduction velocity and an X-linked mutation of connexin 32, consistent with Charcot-Marie-Tooth (CMT) disease. The neuropathy and gait disorder result because connexin is an important component of which of the following? a. Gap junction b. Microtubule c. Sarcoplasmic reticulum d. Sodium channel e. Synaptic vesicle

the answer is a

52. A 27-year-old man presents to the emergency department with asthmatic bronchitis that started 3 days ago. He is given an aerosolized bronchodilator treatment, which relieves his symptoms. Arterial blood gases following bronchodilator therapy demonstrate metabolic acidosis with a normal anion gap. These findings can be attributed to which of the following? a. A decrease in plasma bicarbonate caused by renal compensation for the respiratory alkalosis that existed before treatment b. A decrease in plasma chloride concentration resulting from the chloride shift after the treatment restored alveolar ventilation c. An increase in citrate from the vehicle used in the bronchodilator preparation d. An increase in lactic acid secondary to the hypoxemia that existed before treatment e. A laboratory error

the answer is a

7. A 42-year-old woman consults a dermatologist to evaluate and treat the frown lines on her forehead just above the nose. After the treatment options are explained to her, the patient asks the dermatologist to administer botulinum type A (Botox). Botox smooths out glabellar lines by which of the following mechanisms? a. Blocking the release of synaptic transmitter from α-motoneurons b. Decreasing the amount of calcium released from the sarcoplasmic reticulum c. Enhancing the enzymatic hydrolysis of acetylcholine at the neuromuscular junction d. Increasing the flow of blood into facial muscles e. Preventing the opening of sodium channels on muscle membranes

the answer is a

37. A 23-year-old woman is admitted to the hospital with a 3-month history of malaise and generalized muscle cramps. Laboratory results reveal serum sodium of 144mmol/L, serum potassium of 2.0 mmol/L, serum bicarbonate of 40 mmol/L, and arterial pH of 7.5. Which of the following is the most likely cause of this patient's hypokalemic alkalemia? a. Diabetes b. Hyperaldosteronism c. Hyperventilation d. Persistent diarrhea e. Renal failure

the answer is b

4. A 48-year-old woman with advanced breast cancer presents with severe nausea, vomiting, and dehydration. She is not undergoing chemo-therapy currently. Laboratory findings reveal elevated serum-ionized calcium. Parathyroid hormone (PTH) levels are undetectable, but there is an increase in PTH-related peptide (PTHrP). The increased flow of calcium into the cell is an important component of the upstroke phase of the action potential in which of the following? a. Cardiac ventricular muscle b. Intestinal smooth muscle c. Nerve cells d. Presynaptic nerve terminals e. Skeletal muscle

the answer is b

40. A 64-year-old man with a long history of type 2 diabetes mellitus presents at his internist's office with a chief complaint of weakness and fatigue. Serum chemistries are as follows: Na+, 130 mEq/L; K+, 6.3 mEq/L; HCO3−, 18 mEq/L; BUN, 43 mg/dL; creatinine, 2.9 mg/dL; and glucose, 198mg/dL. The only medication the patient is currently taking is 5 mg glyburide twice daily. These electrolyte and acid-base disturbances are most likely the result of which of the following? a. Hyperreninemia b. Hypoaldosteronism c. Hypocalcemia d. Hypoventilation e. Hypovolemia

the answer is b

45. A patient comes into the emergency department exhibiting signs of hyperkalemia. The extracellular potassium of a hyperkalemic patient can be decreased by administering which of the following drugs? a. Atropine b. Epinephrine c. Glucagon d. Isotonic saline e. Lactic acid

the answer is b

8. A 48-year-old executive was referred for a life insurance physical examination for his new corporation. His body mass index was 34, indicating clinical obesity, and his blood pressure was 145/92 mm Hg. Blood tests showed hyperlipidemia and hyperglycemia with normal insulin levels, consistent with type 2 diabetes mellitus (T2DM). T2DM adversely affects many cellular processes. Which of the following transport processes is a passive downhill process? a. Calcium into the sarcoplasmic reticulum (SR) b. Glucose into skeletal muscle and fat cells c. Hydrogen into the lumen of canaliculi of the parietal cells of the stomach d. Phosphate into epithelial cells lining the proximal tubule of the kidney e. Sodium out of brain cells

the answer is b

12. A 23-year-old man is brought to the emergency department after collapsing during basketball practice. On admission, he is lethargic and appears confused. His coach reports that it was hot in the gym and he was drinking a lot of water during practice. An increase in which of the following is the most likely cause of his symptoms? a. Intracellular tonicity b. Extracellular tonicity c. Intracellular volume d. Extracellular volume e. Plasma volume

the answer is c

13. A 14-year-old adolescent girl reports blood in her urine 2 weeks after she had a sore throat. She has uremia and a blood pressure of 160/90 mm Hg with peripheral edema, suggestive of volume expansion secondary to salt and water retention. Which of the following is the approximate extracellular fluid volume of a normal individual? a. 5% of body mass b. 10% of body mass c. 20% of body mass d. 40% of body mass e. 60% of body mass

the answer is c

2. A 40-year-old male with acute promyelocytic leukemia is started on treatment with all-trans retinoic acid (ATRA). Which of the following characteristics of a hydrophobic hormone that binds to nuclear receptors is most important in governing its diffusibility through a cell membrane? a. Diameter b. Electrical charge c. Lipid solubility d. Molecular weight e. Three-dimensional shape

the answer is c

53. A group of medical students in the Wilderness Medicine Club leave after their exam for a hiking trip in the Rocky Mountains. After spending the night in a hotel in Denver, CO, they drive to the base camp (10,000 ft) the following day, where the guide decides to camp for the night to better acclimate to the altitude. Three of the students grow impatient and announce that they are going ahead to Mt. Elbert, the highest mountain in Colorado (14,400-ft altitude, barometric pressure = 447 mm Hg). About 3 hours later, one of the students returns in a panic to get medical help because his friends are disoriented, ataxic, short of breath, and vomiting. The guide calls for the search and rescue helicopter, which locates the hikers and takes them to the nearest emergency department. A diagnostic workup will likely show a decrease in which of the following values? a. 2,3-Bisphosphoglycerate b. Erythropoietin c. PaCO2 d. pH e. Pulmonary vascular resistance

the answer is c

56. A 25-year-old man, who is a fourth-year medical student at Louisiana State University School of Medicine in New Orleans, LA, enrolls in a month-long clinical elective at the University of Colorado School of Medicine in Denver, CO. Which of the following values will return toward normal after the student has acclimatized to the change in altitude? a. Alveolar ventilation b. Arterial oxygen tension c. Cardiac output d. Hemoglobin concentration e. Plasma bicarbonate concentration

the answer is c

6. A 2-day-old infant starts having brief tonic-clonic seizures throughout the day. His neurological function in between seizures is normal, and he has no other medical or neurological problems. The history reveals no readily apparent causes for the seizures, though the mother recalled that her first baby also developed seizures shortly after birth that only lasted for 2 weeks, with no subsequent episodes or developmental problems. Genetic analysis revealed a mutation of voltage-gated K+ channels consistent with a diagnosis of benign familial neonatal seizures. Which of the following would cause an immediate reduction in the amount of potassium leaking out of a cell? a. Decreasing the extracellular potassium concentration b. Decreasing the extracellular sodium concentration c. Hyperpolarizing the membrane potential d. Increasing the permeability of the membrane to potassium e. Reducing the activity of the sodium-potassium pump

the answer is c

15. A 49-year-old man in end-stage renal failure is able to perform peritoneal dialysis at home. The osmolality of the solution chosen for peritoneal dialysis will determine the rate of ultrafiltration. Which of the following statements best characterizes a molecule whose osmolality is zero? a. It will not permeate the membrane. b. It can only cross the membrane through the lipid bilayer. c. It causes water to flow across the membrane. d. It is as diffusible through the membrane as water. e. It is transported across the membrane by a carrier.

the answer is d

19. A 65-year-old man being treated with a beta-blocker and an angiotensin-converting enzyme (ACE) inhibitor for his heart failure presents to his cardiologist's office complaining of fatigue, weakness, shortness of breath, and an irregular heartbeat. An electrocardiogram (ECG) reveals atrial fibrillation, so his cardiologist adds digoxin to his treatment regimen, but tells the patient that he will need to get his blood drawn to check for low K+ on a regular basis. Hypokalemia will increase the risk and severity of digitalis toxicity because of which of the following? a. Hypopolarization of cardiac muscle membranes b. Increased amplitude of cardiac muscle action potentials c. Increased excitability of cardiac muscle cells d. Increased inhibition of the Na+ − K+pump e. Increased removal of cardiac cytosolic Ca 2+ via the Na+ − Ca2+ exchanger

the answer is d

21. A 55-year-old obese man with type 2 diabetes mellitus presents for his annual checkup. Serum lipoprotein analysis done after a 12-hour fast shows elevated low- density lipoprotein (LDL) and very low-density lipoproteins (VLDL), elevated triglycerides, and decreased high-density lipo-protein cholesterol. What contributes to the lipid abnormalities in type 2 diabetes mellitus? a. Hyperglycemia increases triglyceride uptake into adipose tissue. b. Insulin resistance increases triglyceride uptake into adipose tissue. c. Insulin resistance and hyperglycemia decrease fatty acid flux to the liver and lipolysis. d. Insufficient insulin action in adipose tissue decreases lipoprotein lipase activity. e. Insufficient insulin action increases LDL receptor activity.

the answer is d

28. A 22-year-old man is planning to run a marathon when he goes to visit his brother in Denver, CO. Because of the high altitude, he decides to leave early for Denver to train for the event. While in Denver, he visits an urgent care center after experiencing extensive spasms and cramping in his calf muscles while running, symptoms that he seldom experienced at sea level. Laboratory analysis reveals hypocalcemia. Which of the following is the reason high altitude predisposes to tetany? a. Low oxygen tension causes a decrease in skeletal muscle blood flow b. Low oxygen tension causes an increase in skeletal muscle lactate c. Plasma protein concentration is reduced by hypoxia d. Plasma proteins are more ionized under alkalotic conditions, which provide more protein anion to bind with Ca2+ e. Stimulation of Na+/K+ adenosine triphosphatase (ATPase) reduces the plasma concentration of free ionized Ca2+

the answer is d

55. A 28-year-old pregnant woman develops placental insufficiency at 27 weeks' gestation. It is determined that a preterm caesarian section will be required. To determine how soon the delivery can be done, a sample of amniotic fluid is aspirated. Measurement of the lecithin/sphingomyelin (L/S) ratio in amniotic fluid assesses which of the following? a. Fetal adrenal function b. Fetal brain development c. Fetal kidney development d. Fetal lung maturity e. Placenta gas exchange

the answer is d

14. A previously well 18-year-old woman is admitted to the ICU because of altered mental status. She does not respond to instructions and her arms are postured in a flexor position. Laboratory data reveal a serum sodium concentration of 125 mmol/L. Her friends indicate that the patient had taken ecstasy at a party the night before, and because she was extremely thirsty the next morning, she had consumed a lot of water in a short period of time. Assuming that the reduction in osmolarity is entirely due to water consumption, that her initial weight was 60 kg, and that her initial osmolarity was 300 mOsm/L, which of the following is approximately the quantity of water she would have drunk to produce the observed hyponatremia? a. 2.5 L b. 3.5 L c. 5 L d. 6 L e. 7 L

the answer is e

16. A 76-year-old woman with a history of uncontrolled hypertension presents in the emergency department with hypotension and shock-like symptoms. Her daughter reports systolic blood pressure near 200 mm Hg earlier in the day and suspects a dissecting aneurysm, which is confirmed with a computed tomography (CT) of the chest. Biopsy of the repaired aorta shows giant cell arteritis, and the woman is placed on a regimen of high-dose prednisone. The anti-inflammatory effect of exogenous glucocorticoids is thought to be due to which of the following? a. Activation of phospholipase A2 b. Increased capillary membrane permeability c. Increased formation of leukotrienes d. Increased release of interleukin-1 (pyrogen) from granulocytes e. Inhibition of the activation of nuclear factor-κB (NF-κB)

the answer is e

17. A 62-year-old man presents to the emergency room with an acute onset of aphasia and hemiparesis. A CT scan reveals an increase in intracranial fluid. Which of the following solutions will be most effective in reducing intracranial pressure (ICP) following a large hemispheric stroke? a. 150 mmol sodium chloride b. 250 mmol glycerol c. 250 mmol glucose d. 350 mmol urea e. 350 mmol mannitol

the answer is e

30. A 78-year-old woman recovering from a stroke is observed to have altered mental status. She is taken from the nursing home to the emergency department, where physical examination shows signs of dehydration and laboratory test results show a blood glucose concentration of 600 mg/dL and plasma osmolarity of 340 mOsm/L. Which of the following physiological variables is likely increased in this patient? a. Arterial pH b. Alveolar PaCO2 c. Intracellular volume d. Plasma sodium concentration e. Urine volume

the answer is e

36. An 84-year-old woman presents with muscle weakness, cramping, irritability, and neuromuscular excitability. Electrolytes reveal hypokalemia and a higher-than-normal plasma bicarbonate concentration. Which of the following conditions causes metabolic alkalosis? a. Diarrhea b. Hypoaldosteronism c. Hypoxemia d. Renal failure e. Treatment with a loop diuretic

the answer is e

41. A 60-year-old man presents with symptoms of vitamin B12 deficiency. Further diagnostic tests reveal that he has pernicious anemia. The underlying problem in pernicious anemia is which of the following? a. Autoimmune destruction of chief cells in the gastric mucosa b. Gluten-induced enteropathy c. Inadequate dietary intake of cyanocobalamin d. Infection with Diphyllobothrium latum e. Lack of intrinsic factor

the answer is e

42. An 89-year-old nursing home resident exhibits signs of vitamin deficiency. At concentrations present in the diet, which of the following vitamins is absorbed primarily by diffusion? a. Folate b. Niacin c. Vitamin B12 d. Vitamin C e. Vitamin D

the answer is e

48. A 49-year-old man is brought to the emergency department with weakness, confusion, and shortness of breath. The ECG reveals QRS complex widening and flattened P waves typical of serum potassium concentrations exceeding 7.5 mEq/L. Which of the following conditions results in hyperkalemia? a. Adrenal medullary stimulation b. Diuretic therapy c. Insulin administration d. Metabolic alkalosis e. Volume depletion

the answer is e


Related study sets

UD1: Metodología de investigación en Psicología

View Set

Inquizitive Chapter 04. Civil Liberties

View Set

oncological disorders of pediatric

View Set